Προβλήματα Μαθηματικών


Τεύχη του ηλεκτρονικού περιοδικού "Εικοσιδωδεκάεδρο"

Όλα τα τεύχη του ηλεκτρονικού περιοδικού "Εικοσιδωδεκάεδρο" με πολύ ενδιαφέροντα προβλήματα Μαθηματικών μπορείτε να τα βρείτε στην ακόλουθη διεύθυνση


Μέχρι σήμερα έχουν εκδοθεί 11 τεύχη 
________________________________________________


__________________________________________________________

Προβλήματα που έχω αναρτήσει
στο Φόρουμ www.mathematica.gr


1. Σάββατο 15 Ιανουαρίου 2011
Με αφορμή το 3ο θέμα του διαγωνισμού Ευκλείδης 2011 της ΕΜΕ για την Α Λυκείου,
θέτω για απόδειξη το παρακάτω κριτήριο.
Ένα παραλληλόγραμμο, στο οποίο μία διαγώνιός του είναι διχοτόμος μιας γωνίας του, είναι ρόμβος.

2. Δευτέρα 31 Ιανουαρίου 2011
Δίνεται ένα επίπεδο Ρ και ένα σημείο του έστω Α.
Αν Β ένα σημείο εκτός του επιπέδου Ρ, να βρεθούν τα σημεία Χ του Ρ για τα οποία ο λόγος \frac{AB + AX}{BX} παίρνει τη μέγιστη τιμή του.
Να διακρίνετε δύο περιπτώσεις:
1. Το τμήμα ΑΒ είναι κάθετο στο Ρ.
2. Το τμήμα ΑΒ δεν είναι κάθετο στο Ρ.

3. Τρίτη 9 Νοεμβρίου 2010
Σε μία πόλη στις δημοτικές εκλογές του έτους α, ψήφισαν 10000 άτομα, υπήρχαν οι συνδυασμοί Α, Β, Γ, Δ
με ψήφους και ποσοστά, όπως φαίνονται στον πίνακα που ακολουθεί:
Α: ψήφοι 4000, ποσοστό 40%
Β: ψήφοι 3000, ποσοστό 30%
Γ: ψήφοι 2000, ποσοστό 20%
Δ: ψήφοι 1000, ποσοστό 10%

Το έτος α + 4 στην ίδια πόλη ψήφισαν 8000 άτομα (υπήρχε αποχή), πάλι υπήρχαν οι τέσσερις συνδυασμοί Α, Β, Γ, Δ
με ψήφους και ποσοστά, όπως φαίνεται στον παρακάτω πίνακα:
Α: ψήφοι 3000, ποσοστό 37,5%
Β: ψήφοι 2500, ποσοστό 31,25%
Γ: ψήφοι 2000, ποσοστό 25%
Δ: ψήφοι 500, ποσοστό 6,25%
Οι συνδυασμοί Β και Γ πανηγύριζαν, αφού αύξησαν τα ποσοστά τους, ο Β από 30% σε 31,25% και ο Γ από 20% σε 25%.
Ένας μαθητής ρώτησε, πώς γίνεται αυτό; μπορεί να χάνεις σε ψήφους και να αυξάνεται το ποσοστό σου σε σχέση με τις προηγούμενες εκλογές;
Μία μαθήτρια ρώτησε, πώς γίνεται να έχεις ακριβώς τους ίδιους ψήφους σε σχέση τις προηγούμενες εκλογές και τώρα να αυξάνεται το ποσοστό σου;
Να μία άσκηση για να προβληματιστούν μαθητές των τελευταίων τάξεων του Δημοτικού και των πρώτων τάξεων του Γυμνασίου, διότι υποθέτω ότι η απάντηση σε τέτοια ερωτήματα για ενήλικες είναι "προφανής".

4. Δευτέρα 7 Φεβρουαρίου 2011

Δεν γνωρίζω αν το θέμα έχει συζητηθεί παλαιότερα.
Να εξεταστεί αν υπάρχει μονότονη και συνεχής συνάρτηση στο R,
για την οποία να ισχύει: x^{2} \leq f(x) \leq x^{2} + 1 , για κάθε πραγματικό αριθμό x.5.

5. Δευτέρα 31 Ιανουαρίου 2011

Με αφορμή το θέμα που τέθηκε εδώ  viewtopic.php?f=50&t=10783
διατυπώνω το εξής κριτήριο για ισόπλευρα τρίγωνα.
Αν στο οξυγώνιο τρίγωνο ΑΒΓ φέρουμε τις εσωτερικές διχοτόμους ΑΔ, ΒΕ, ΓΖ, οι οποίες τέμνονται στο σημείο Ο και ισχύει (ΑΖΟ) = (ΒΟΔ) = (ΓΟΕ), τότε το ΑΒΓ είναι τρίγωνο ισόπλευρο.

6. Δευτέρα 31 Ιανουαρίου 2011

Με αφορμή το θέμα που έθεσε ο Θ. Μάγκος εδώ viewtopic.php?f=50&t=10783, διατυπώνω ένα αντίστοιχο ερώτημα:
Στο οξυγώνιο τρίγωνο ΑΒΓ, φέρουμε τα ύψη ΑΔ, ΒΕ, ΓΖ, τα οποία τέμνονται στο Ο.
Αν ισχύει (ΑΖΟ) = (ΕΟΓ) ή (ΕΟΓ) = (ΟΔΒ) ή (ΟΔΒ) = (ΑΖΟ), τότε το τρίγωνο ΑΒΓ είναι ισοσκελές;

7. Πέμπτη 27 Ιανουαρίου 2011

Αν x, α, β, γ > 0, τότε το παρακάτω σύστημα
x(x + α) = β(x + γ)
x(x + β) = γ(x + α)
x(x + γ) = α(x + β)
έχει και άλλες λύσεις εκτός από την x = α = β = γ;

8. Τρίτη 25 Ιανουαρίου 2011

Έχουμε μία ζυγαριά με δύο δίσκους.
Πόσα ζύγια (ακέραιου αριθμού κιλών) χρειαζόμαστε για να ζυγίσουμε
οποιοδήποτε αντικείμενο (πάλι ακέραιου αριθμού κιλών) από 1 έως και 20 κιλά; Εννοείται ότι πρέπει να βρούμε τον μικρότερο αριθμό από τα απαραίτητα ζύγια.

9. Κυριακή 23 Ιανουαρίου 2011

Είναι θέμα που το έδωσε ο Νίκος Ζανταρίδης,
Να αποδειχθεί ότι δεν υπάρχει συνεχής συνάρτηση f: R --> R,
για την οποία να ισχύει \left|f(x) - f(y) \right|\geq \sqrt{\left|x-y \right|} για κάθε x, y.

10. Σάββατο 22 Ιανουαρίου 2011

Δίνεται ο κύκλος (Κ, R) με συντεταγμένες του σημείου Κ, (R, R) και R >0.
Από σημείο Α του (Κ, R) φέρουμε την εφαπτομένη ευθεία, η οποία τέμνει τον άξονα xx' στο Β. Ονομάζουμε Γ την προβολή του Α στον άξονα yy'.
Αν Μ το μέσο του ΒΓ, να βρεθεί ο γεωμετρικός τόπος των μέσων M,
όταν το Α κινείται στον (Κ, R).
Αν υπάρχει ενδιαφέρον για το θέμα, θα δώσω και μία πρόχειρη εικόνα του γεωμετρικού τόπου. Στην υγειά του Geogebra που "μου άνοιξε τα μάτια".

11. Σάββατο 22 Ιανουαρίου 2011

Αν για τους πραγματικούς αριθμούς x_{i}, y_{i}, i= 1, 2, 3 ισχύει:

x^{2}_{1} + y^{2}_{1} = x^{2}_{2} + y^{2}_{2} = x^{2}_{3} +y^{2}_{3}=k^{2} > 0, και τα διατεταγμένα ζεύγη (xi , yi) δεν ταυτίζονται,

τότε η παράσταση Α =\left|\left(x_{3}-x_{1} \right)\left(y_{2}-y_{1} \right)-\left(x_{2}-x_{1})\left(y_{3}-y_{1} \right) \right|
παίρνει τη μέγιστη τιμή της, όταν ισχύει:

\left(x_{1}-x_{2} \right)^{2} + \left(y_{1}-y_{2} \right)^{2} = \left(x_{1}-x_{3} \right)^{2} + \left(y_{1} -y_{3}\right)^{2} = \left(x_{2}-x_{3} \right)^{2}+\left(y_{2}-y_{3} \right)^{2}.

Επίσης, να δοθεί γεωμετρική ερμηνεία των ενεργειών και των συμπερασμάτων σας.

12. Πέμπτη 20 Ιανουαρίου 2011

Παρουσιάζω ένα θέμα που έχει προκύψει από τη σχολική τάξη.
Δίνεται κύκλος (Ο, ρ), μία διάμετρός του ΑΒ και σημείο Γ της ευθείας ΑΒ προς το μέρος του Β ώστε ΒΓ = 3ρ.
1) Να βρεθεί η δύναμη του σημείου Γ ως προς τον (Ο, ρ).
2) Να αποδειχθεί ότι υπάρχει σημείο Δ του (Ο, ρ), ώστε ΓΔ = 2ρ.
3) Να αποδειχθεί ότι η ευθεία ΓΔ τέμνει τον (Ο, ρ) και σε ένα άλλο σημείο έστω Ε.
4) Να βρεθεί το μήκος του ΔΕ.
Σημείωση: Το διδακτικό ενδιαφέρον βρίσκεται στο ερώτημα 4).
Να γραφούν σχόλια σχετικά με την απάντησή σας στο ερώτημα αυτό.

13. Τρίτη 11 Ιανουαρίου 2011

Να βρεθεί ο ελάχιστος αριθμός ζυγίσεων που απαιτούνται
(σε ζυγαριά με δύο δίσκους, που δέχονται "μεγάλα" βάρη)
για να εντοπίσουμε μία σφαίρα, που μόνο αυτή είναι βαρύτερη, από ένα πλήθος ν σφαιρών.

14. Τετάρτη 19 Ιανουαρίου 2011
Ως συνέχεια του θέματος που είχα αναρτήσει εδώ: viewtopic.php?f=49&t=12415, δίνω το εξής πρόβλημα.

Από ένα σύνολο ν σφαιρών, δύο είναι βαρύτερες.
Να βρεθεί ο ελάχιστος αριθμός ζυγίσεων (με ζυγό με δύο δίσκους)
που απαιτούνται για τον εντοπισμό των δύο βαρύτερων σφαιρών.

Διευκρίνιση: Οι δύο βαρύτερες σφαίρες έχουν την ίδια μάζα.

15. Δευτέρα 17 Ιανουαρίου 2011

Αν x, y, z, n, φυσικοί αριθμοί με n > 1 και ισχύει  x^{n}+y^{n}=z^{n},  τότε να αποδειχθεί ότι οι αριθμοί x, y, z είναι μεγαλύτεροι από τον n.
Σημειώση: Δεν είναι κατασκευή δική μου, την αντέγραψα από βιβλίο.

16. Σάββατο 15 Ιανουαρίου 2011

Με αφορμή το 3ο θέμα του διαγωνισμού Ευκλείδης 2011 της ΕΜΕ για την Γ Λυκείου, θέτω για απόδειξη το ακόλουθο κριτήριο.

Αν οι κύκλοι (Κ, ρ) και (Λ, R) έχουν κοινή χορδή την ΑΒ,
για ένα σημείο Γ του (Κ, ρ) και ένα σημείο Δ του (Λ, R)
ισχύει γωνία ΑΓΒ = γωνία ΑΔΒ, τότε οι κύκλοι είναι ίσοι.


17. ...................

18. Πέμπτη 6 Ιανουαρίου 2011

Το θέμα που προτείνω δεν επιχείρησα να το λύσω,
διότι με το Geogebra "είδα" ότι το ζητούμενο σημείο κινείται σε μία άγνωστη καμπύλη.
Είναι μία (μερική) γενίκευση των προβλημάτων που έθεσα http://www.mathematica.gr/forum/viewtopic.php?f=20&t=11890 και
http://www.mathematica.gr/forum/viewtopic.php?f=22&t=11967.

Δίνονται δύο ίσοι κύκλοι (Κ, ρ) και (Λ, ρ) με ΚΛ > 2ρ
και ένα σημείο Α στη μεσοκάθετο του ΚΛ.
Να βρεθεί ο γεωμετρικός τόπος των κορυφών Δ του ορθογωνίου ΑΒΓΔ,
όταν το Β ανήκει στον (Κ, ρ) και το Γ ανήκει στον (Λ, ρ).


19. Δευτέρα 10 Ιανουαρίου 2011

Αν η σύνθεση fofof είναι συνεχής για κάθε x πραγματικό,
μπορεί να fof να είναι ασυνεχής σε κάποιο xο;

20. Δευτέρα 10 Ιανουαρίου 2011

Να αποδειχθεί ότι ο αριθμός 1 + n + n^{2}+n^{3} + n^{4}
δεν διαιρείται από τον 4 για καμία τιμή του φυσικού n.

Το θέμα αυτό προτάθηκε από τον Ivan Matic για τον διαγωνισμό ΒΑΜΟ
κατά την προετοιμασία στο Berkeley Mathematics Circles.
Το προτείνουμε για την προετοιμασία όσων μαθητών θα συμμετέχουν
στον διαγωνισμό "Ευκλείδη 2010" το Σάββατο 15 Ιανουαρίου 2011.

21. Κυριακή 9 Ιανουαρίου 2011

Αν για τις συνεχείς συναρτήσεις f και g ισχύει f(g(x)) = 0 και g(f(x)) = 0,
για κάθε x πραγματικό αριθμό, τότε τουλάχιστον μία από αυτές τις συναρτήσεις είναι η μηδενική;

22. Σάββατο 8 Ιανουαρίου 2011

Αν για τις συναρτήσεις f και g ισχύουν: f(g(x)) = x και g(f(x)) = x, για κάθε x πραγματικό αριθμό, τότε οι f, g είναι μεταξύ τους αντίστροφες;

23. Κυριακή 9 Ιανουαρίου 2011

Αν για τις συναρτήσεις f και g ισχύει f(g(x)) = 0, για κάθε x πραγματικό αριθμό, τότε τουλάχιστον μία από αυτές τις συναρτήσεις είναι η μηδενική;

24. Παρασκευή 7 Ιανουαρίου 2011

Αν για τις συναρτήσεις f και g ισχύει f(g(x)) = x, για κάθε x πραγματικό αριθμό, τότε οι f, g είναι μεταξύ τους αντίστροφες;

25. Παρασκευή 7 Ιανουαρίου 2011

Να αποδειχθεί ή να απορριφθεί η εξής εικασία:
Αν οι συναρτήσεις f και g είναι 1-1 και ισχύει f(g(x)) = g(f(x)) για κάθε x πραγματικό αριθμό, τότε οι συναρτήσεις αυτές είναι μεταξύ τους αντίστροφες.

26. Πέμπτη 6 Ιανουαρίου 2011

Για το πρόβλημα της 4ης κορυφής ενός ορθογωνίου.
Το κείμενο αυτό συνοψίζει και επεκτείνει τη συζήτηση και τα συμπεράσματα που προέκυψαν μετά την ανάρτηση των θεμάτων
viewtopic.php?f=20&t=11890 και viewtopic.php?f=22&t=11967
Το πρόβλημα στη γενική του μορφή διατυπώνεται ως εξής:
Δίνεται κύκλος (Κ, ρ) και σημείο Α. Αν τα σημεία Β, Γ ανήκουν στον (Κ, ρ)
και το ΑΒΓΔ είναι ορθογώνιο, να βρεθεί ο γεωμετρικός τόπος του σημείου Δ.
Το πρόβλημα αυτό το έχω δει αρκετές φορές σε προβλήματα γεωμετρικών τόπων.
Δύο πρόσφατες «ματιές» μου έπεσαν στα εξής βιβλία:
«Ευκλείδειος Γεωμετρία» για την Α’ τάξη Λυκείου Θετικής Κατευθύνσεως
του Σπύρου Κανέλλου, ΟΕΔΒ, Αθήνα, 1976, σελίδα 29.
«Straight Lines and Curves» των N. Vasilyev και V. Gutenmacher, εκδόσεις Mir, Μόσχα, 1980, σελίδα 49, πρόβλημα 2.13. Ένα εξαιρετικό βιβλίο για θέματα κινούμενων σχημάτων, στα οποία περιλαμβάνονται και οι αποκαλούμενοι γεωμετρικοί τόποι.

Για το πρόβλημα πρέπει να διακρίνουμε τρεις διαφορετικές περιπτώσεις:
1) Το σημείο Α είναι εξωτερικό του κύκλου (Κ, ρ)
2) Το σημείο Α είναι εσωτερικό του κύκλου (Κ, ρ)
3) Το σημείο Α ανήκει στον κύκλο (Κ, ρ)
Επίσης, για τα σημεία Β και Γ πρέπει να διακρίνουμε δύο διαφορετικές διατάξεις τις ΑΒΓΔ και ΑΒΔΓ για κάθε μία από τις 3 περιπτώσεις.
Στη διαδικασία επίλυσης του προβλήματος συμμετείχαν με λύσεις, ιδέες και προτάσεις οι, p-gianno, rek2, Σωτήρης Λουρίδας και Κώστας Δόρτσιος.
Μάλιστα ο Κώστας πρότεινε την εξής επέκταση του προβλήματος:
Θα μπορούσαμε να ζητήσουμε και το γ.τ. του κέντρου Σ του ορθογωνίου ΑΒΓΔ.
Η λύση στην πρώτη περίπτωση είναι τόξο κύκλου και όχι κύκλος όπως «φαίνεται» με μια πρώτη ματιά. Αυτό συμβαίνει επειδή το σημείο Δ δεν μπορεί να κατέχει όλες τις θέσεις του ομόκεντρου κύκλου προς τον (Κ, ρ).
Η λύση στη δεύτερη περίπτωση είναι το σημείο Δ κινείται σε κύκλο ομόκεντρο του (Κ, ρ) του οποίου την ακτίνα μπορούμε να προσδιορίσουμε αξιοποιώντας το 1ο θεώρημα των διαμέσων ενός τριγώνου. Ακόμα και όταν τα σημεία Β και Γ έχουν διαφορετική διάταξη στο ορθογώνιο, πάλι η λύση είναι κύκλος ομόκεντρος προς τον αρχικό.
Στην τρίτη περίπτωση που είναι η πιο απλή, το σημείο Δ είναι και αυτό σημείο του (Κ, ρ).
Μάλιστα, αν η διάταξη των κορυφών του ορθογωνίου είναι ΑΒΓΔ, τότε το Δ είναι το αντιδιαμετρικό οποιουδήποτε σημείου του (Κ, ρ) θεωρήσουμε ως Β.
Αν η διάταξη είναι ΑΒΔΓ, τότε το Δ είναι το αντιδιαμετρικό του Α, δηλαδή είναι μοναδικό.
Τι το ενδιαφέρον έχει το πρόβλημα αυτό από την άποψη της διδασκαλίας της Γεωμετρίας;
Για την Α’ Λυκείου οι περιπτώσεις 1η και 3η είναι κατάλληλες για μελέτη από τους μαθητές της τάξης αυτής,
αφού οι γνώσεις που απαιτούνται είναι οι ιδιότητα της μεσοκαθέτου, ο ορισμός του κύκλου, οι ιδιότητες του ορθογωνίου. Επιπλέον, αυτό είναι το βασικό χαρακτηριστικό του προβλήματος,
οι μαθητές πρέπει να γίνουν προσεκτικοί ώστε να διακρίνουν ακραίες περιπτώσεις, περιορισμούς στο πρόβλημα. Επίσης, ο διδάσκων μπορεί να θέσει την περίπτωση 2η, η οποία δεν αντιμετωπίζεται από μαθητές της Α Λυκείου, λόγω του θεωρήματος των διαμέσων που πρέπει να γνωρίζουν,
ως δείγμα ότι ένα πρόβλημα με μία τροποποίηση των δεδομένων δυσκολεύει δραματικά.
Επίσης, για τους μαθητές της Θετικής και Τεχνολογικής Κατεύθυνσης της Β΄ Λυκείου, το θέμα έχει ενδιαφέρον να αντιμετωπιστεί και με διανυσματικό τρόπο και να συγκριθεί η τεχνική αυτή,με αυτές της Ευκλείδειας Γεωμετρίας. Ας σημειωθεί ότι ένα τέτοιο πρόβλημα περιέχει και «οριακές καταστάσεις», όπως επεσήμανε ο Σ. Λουρίδας, για παράδειγμα, τι θα γίνει αν το σημείο Β πλησιάζει πολύ κοντά στο Α, ή συμπέσει με αυτό;
Ιδιαίτερη διδακτική αξία έχει ένα τέτοιο πρόβλημα, διότι θέτει νέα ερωτήματα με ελαφρές έως ριζικές τροποποιήσεις των δεδομένων και των ζητουμένων του, όπως έδειξε ο Κ. Δόρτσιος (και θα κάνω κι εγώ), και κυρίως διότι μπορεί να μπλέξει στο «παιχνίδι της έρευνας» και μαθητές που δεν έχουν καλό μαθηματικό υπόβαθρο, αλλά μπορούν να χειριστούν υπολογιστικά εργαλεία, όπως το Geogebra, Cabri, ή ακόμα κι αν δεν έχουν ιδέα από όλα αυτά, αρκεί να έχουν το ενδιαφέρον να παρακολουθήσουν τη διαδικασία «ψαξίματος» της λύσης του προβλήματος σε μία οθόνη όταν ο διδάσκων ή κάποιος συμμαθητής τους χειρίζεται τα εργαλεία του προγράμματος.
Τέτοιου είδους προβλήματα δείχνουν πώς λειτουργούν και εμπνέονται οι κατασκευαστές μαθηματικών προβλημάτων, πώς δοκιμάζουν με υπολογιστικά μέσα τις εικασίες τους και όταν βεβαιωθούν για αυτές δίνουν «το οριστικό κτύπημα», δηλαδή την απόδειξη.
Θέτω και εγώ με τη σειρά μου ένα πρόβλημα σαφώς δυσκολότερο,
το οποίο θα αναρτήσω σύντομα.

27. Κυριακή 19 Δεκεμβρίου 2010

Να βρείτε όλους τους πρώτους φυσικούς αριθμούς α, β και γ για τους οποίους ισχύει:   αβ + βγ + αγ > αβγ.
Την πηγή από όπου πήρα το πρόβλημα, θα την αναρτήσω μετά την επίλυση της άσκησης από τους ενδιαφερόμενους νεαρούς λύτες.

28. Κυριακή 26 Δεκεμβρίου 2010

Ως συνέχεια του θέματος http://www.mathematica.gr/forum/viewtopic.php?f=20&t=11890 προτείνω
το εξής θέμα:
Δίνεται κύκλος (Κ, ρ) και σημείο Α στο εσωτερικό του.
Αν τα σημεία σημεία Β, Γ ανήκουν στον (Κ, ρ) και το ΑΒΓΔ είναι ορθογώνιο, να βρεθεί ο γεωμετρικός τόπος των σημείων Δ.

Μετά τη δημοσίευση λύσεων στο θέμα αυτό, θα κάνω μία σειρά παρατηρήσεων, διάκρισης περιπτώσεων και σχετικών σχολίων.

29. Τετάρτη 22 Δεκεμβρίου 2010

Δίνεται ένα τρίγωνο ΑΒΓ. Να κατασκευαστεί τετράγωνο του οποίου οι κορυφές είναι σημεία του τριγώνου ΑΒΓ.  (Εγγεγραμμένο τετράγωνο σε τρίγωνο).

30. Τετάρτη 22 Δεκεμβρίου 2010

Δίνεται το πολυώνυμο
P(x) = x^{4}+ (a-1)x^{3} - (2a^{2}+1)x^{2}+ (a^{2}+1)x + a^{2}-a.
Να αποδειχθεί ότι για κάθε τιμή της παραμέτρου a, το πολυώνυμο έχει πραγματικές ρίζες.

31. Δευτέρα 20 Δεκεμβρίου 2010

Να αποδειχτεί ότι δεν υπάρχει τρίγωνο στο οποίο ο εγγεγραμμένος κύκλος
τριχοτομεί μία διχοτόμο του τριγώνου.


32. Πέμπτη 16 Δεκεμβρίου 2010
Στο συνημμένο σχήμα έχουμε ορθοκανονικό σύστημα αξόνων και ΟΒ = 10.
Το σημείο Γ κινείται στον άξονα xx', ώστε 0 < ΟΓ < 10.
Αν Δ το ίχνος της διχοτόμου της γωνίας ΒΓΟ στον άξονα yy',
να βρεθεί η απόσταση ΟΔ για τις διάφορες θέσεις του σημείου Γ.

33. Τετάρτη 15 Δεκεμβρίου 2010

Με αφορμή το θέμα που δημοσιεύθηκε εδώ http://www.mathematica.gr/forum/viewtopic.php?f=23&t=11535,  να αποδειχθεί με τη χρήση του Διανυσματικού Λογισμού το αντίστροφο.
"Αν σε ένα τρίγωνο ΑΒΓ το ΑΔ είναι ύψος του και ισχύει
 \overrightarrow{{\rm{A}}\Delta} ^{2} = \overrightarrow{{\rm{B}}\Delta}\cdot\overrightarrow{\Delta \Gamma} ,  τότε το τρίγωνο είναι ορθογώνιο με ορθή γωνία την Α".

34. Πέμπτη 16 Δεκεμβρίου 2010

Δίνεται το ισόπλευρο τρίγωνο ΑΒΓ και τα κανονικά πεντάγωνα ΑΓΔΕΖ και ΑΗΘΙΒ, τα οποία έχουν κοινά σημεία με το τρίγωνο ΑΒΓ μόνο αυτά των πλευρών ΑΓ και ΑΒ αντίστοιχα.
1) Να εξεταστεί αν ΗΒ // ΖΓ.
2) Να εξεταστεί αν τα σημεία Θ, Α, Δ είναι συνευθειακά
3) Να εξεταστεί αν ΘΕ // ΒΓ.
4) Να εξεταστεί αν το τετράπλευρο ΗΖΓΒ είναι εγγράψιμο σε κύκλο.

35. Πέμπτη 9 Δεκεμβρίου 2010

Με αφορμή το θέμα που τέθηκε http://www.mathematica.gr/forum/viewtopic.php?f=52&t=11461,
θέτω ένα σχετικό ερώτημα:
Αν δύο συναρτήσεις f, g είναι ασυνεχείς σε κάθε σημείo του R,
τότε και η σύνθεση fog θα είναι και αυτή ασυνεχής στο R;

36. Σάββατο 4 Δεκεμβρίου 2010

Για τη συνεχή συναρτηση f στο [0, 1] ισχύει:
\int_{0}^{x}{f(t)dt} = 1 - \frac{1}{g(x)} για κάθε x του διαστήματος [0, 1]
και η g είναι μία άγνωστη συνάρτηση ορισμένη στο ίδιο διάστημα.
Να αποδειχθεί ότι: g(0) + g(1/2) + g(1) > 0.

37. Σάββατο 4 Δεκεμβρίου 2010

Με αφορμή του θέμα που παρουσιάστηκε εδώ,
http://www.mathematica.gr/forum/viewtopic.php?f=22&t=11119
να κατασκευαστεί τρίγωνο ΑΒΓ με τα εξής δεδομένα:
ΒΓ = 8, γωνία ΒΓΑ = 120 μοίρες και η διάμεσος ΓΔ είναι κάθετη στην πλευρά ΒΓ.

38. Κυριακή 28 Νοεμβρίου 2010

Με αφορμή το θέμα, posting.php?f=51&t=11104, προτείνω μία τροποποίηση που το δυσκολεύει αρκετά.
Η δική μου προσέγγιση απαιτεί και μέγιστα-ελάχιστα με τη βοήθεια των παραγώγων.
Έστω οι μιγαδικοί z με z = (\lambda ^{2}-3) + (2λ+1)i , λεR
1) Να βρείτε τον γεωμετρικό τόπο των εικόνων των μιγαδικών z,
2) Να βρείτε τον μιγαδικό z που η εικόνα του βρίσκεται πλησιέστερα στην αρχή των αξόνων.
Έχει δίκιο ο Κώστας όπως έμμεσα και ευγενικά το τόνισε.
Δεν είναι εύκολο χωρίς τον τύπο επίλυσης τριτοβάθμιων πολυωνυμικών εξισώσεων να βρούμε τη ζητούμενη λύση.
Το ερώτημα 2) έπρεπε να τροποποιηθεί ως εξής:
Να αποδείξετε ότι υπάρχει ένας μοναδικός μιγαδικός z που η εικόνα του βρίσκεται πλησιέστερα στην αρχή των αξόνων.
Εξάλλου, για τον λόγο αυτό είχα αναφέρει και τον εντοπισμό ακροτάτων με τη βοήθεια των παραγώγων.

39. Παρασκευή 26 Νοεμβρίου 2010

Για τους πραγματικούς αριθμούς k_{i} ισχύει 0 < k_{i-1} < k_{i} για κάθε i από 1 έως n.
Να αποδειχθεί ότι η εξίσωση (ως προς τον πραγματικό αριθμό x)
(k_{n})^{x} = (k_{n-1})^{x} + (k_{n-2})^{x} + ...+(k_{2})^{x} + (k_{1})^{x}
έχει μία ακριβώς λύση.

40. Τετάρτη 13 Οκτωβρίου 2010

Σε έναν κύκλο φέρουμε τις κάθετες χορδές ΑΒ και ΓΔ.
Το μέρος του κυκλικού δίσκου που ορίζεται από τις κάθετες χορδές και το έλασσον τόξο ΑΓ έχει εμβαδόν 4 τ.μ.
Το μέρος του κυκλικού δίσκου που ορίζεται από τις κάθετες χορδές και το έλασσον τόξο ΑΔ έχει εμβαδόν 10 τ.μ.
Το μέρος του κυκλικού δίσκου που ορίζεται από τις κάθετες χορδές και το έλασσον τόξο ΒΓ έχει εμβαδόν 8 τ.μ.
Μπορεί να βρεθεί το εμβαδόν του μέρους του κυκλικού δίσκου που ορίζεται από τις κάθετες χορδές και το έλασσον τόξο ΒΔ;

41. Σάββατο 13 Νοεμβρίου 2010

Με αφορμή το θέμα που προτάθηκε εδώ http://www.mathematica.gr/forum/viewtopic.php?f=51&t=10661
να απαντηθούν τα παρακάτω ερωτήματα:
Ποια είναι η σχέση της εικόνας ενός μιγαδικού z και της εικόνας
1. του -z
2. του -\bar{z}
3. του 1/z
4. του -1/\bar{z}
5. του z^{2}
6. του -1/z
7. του \left(\bar{z} \right)^{2}
Καθώς η εκφώνηση ζητάει τη σχετική θέση των εικόνων των μιγαδικών, καταλαβαίνω ότι δεν ενδιαφερόμαστε για τα τεταρτημόρια στα οποία βρίσκονται οι εικόνες.
Ας είναι \displaystyle{z=x+yi} με \displaystyle{x,y \in \mathbb{R}.}
Επειδή \displaystyle{-z=-x-yi}, οι \displaystyle{z,-z} έχουν εικόνες σημεία συμμετρικά ως προς την αρχή του μιγαδικού επιπέδου.
Επειδή \displaystyle{-\bar{z}=-x+yi}, οι \displaystyle{z,-\bar{z}} έχουν εικόνες σημεία συμμετρικά ως προς τον άξονα των φανταστικών.
Για τα υπόλοιπα, χρειαζόμαστε τον μιγαδικό υπό τριγωνομετρική μορφή.
Θάνο,
αυτό ακριβώς ήθελα να τονίσω. Την αξία της χρήσης της τριγωνομετρικής μορφής ενός μιγαδικού για τον πολλαπλασιασμό (ή τη διαίρεση) και κατ΄ επέκταση τη χρήση της στις δυνάμεις μιγαδικών.
Και μάλιστα, όταν αυτό είναι πλήρως κατανοητό από τον καθένα.
Για παράδειγμα. Αν ο z σε τριγωνομετρική μορφή γράφεται: z = \left|z \right| \left(cos(\phi) + isin\phi  \right), τότε \frac{1}{z} = z^{-1}= \left|z \right|^{-1}\left(cos(-\phi) + isin(-\phi) \right).
Γνωρίζουμε ότι οι εικόνες των αριθμών z και -z είναι συμμετρικές ως προς το σημείο (0, 0). Άρα, οι εικόνες των -z και 1/z και (0, 0) είναι σημεία συνευθειακά και η απόσταση του z από το (0, 0) είναι αντιστρόφως ανάλογη με αυτή του 1/z ως προς το (0, 0).
Επίσης, μπορούμε να αναφερθούμε και σε στροφή των εικόνων των z και 1/z γύρω από το (0, 0) με μία μεταβολή της απόστασης τους από αυτό.
Αντίστοιχα, οι θέσεις των εικόνων του z και του z^{2}, σχετίζονται με μία έννοια "στροφής" του δεύτερου προς τα αριστερά με κέντρο στροφής το (0, 0) κατά γωνία φ, όπου φ είναι η γωνία που σχηματίζει η διανυσματική ακτίνα της εικόνας του z σε σχέση με τον άξονα των πραγματικών αριθμών.
Οι τριγωνομετρική μορφή ενός μιγαδικού (πολικές συντεταγμένες) ερμηνεύει (προφανώς) και τη θέση των εικόνων των -z, \bar{z}, κλπ σε σχέση με την εικόνα του z, όπως γνωρίζουμε και με τη χρήση καρτεσιανών συντεταγμένων.
Ειλικρινά, δεν βλέπω κάποιο σοβαρό λόγο να υπάρχουν στοιχεία μιγαδικών αριθμών στην σχολική ύλη, χωρίς την τριγωνομετρική τους μορφή, αφού αυτή είναι που αναδεικνύει την δυναμική της στην επίλυση εξισώσεων και στην εκτέλεση φαινομενικά "δύσκολων" πράξεων.
Συνεπώς, αφού διώξαμε την Τριγωνομετρία (θα γελάσουμε πολύ του χρόνου), δεν βλέπω κάποιον λόγο να κρατήσουμε κάποια στοιχεία μιγαδικών αριθμών.
Δεν προσθέτουν κάτι στην μαθηματική μόρφωση των μαθητών μας. Είναι μία "δήθεν" γνώση χωρίς ουσία και στόχο. Κατανοώ ότι αυτή είναι μία λίγο "ακραία" άποψη, είναι όμως άποψη που κατατίθεται δημόσια και για το λόγο αυτό είναι ειλικρινής.

42. Δευτέρα 27 Σεπτεμβρίου 2010

Με αφορμή την άσκηση του Chris php?f=51&t=9494 για ένα θέμα με μέτρα μιγαδικών δίνω το εξής:
Για κάθε φυσικό n μεγαλύτερο ή ίσο του 2, να αποδειχθεί ότι
 1 \leq  (n-1)sin(\frac{\pi }{n}).

43. Πέμπτη 23 Σεπτεμβρίου 2010

Το θεώρημα αυτό, το βρήκα σε έναν υποφάκελλο στο πρόγραμμα Geometer's Sketchpad, με το τίτλο "Το θεώρημα του Blackwell".
Πράγματι, υπάρχει ως τέτοιο θεώρημα στο βιβλίο του Blackwell - ο τίτλος είναι σε μετάφραση -  "Η Γεωμετρία στην Αρχιτεκτονική" , εκδόσεις Key Curriculum Press, Ιούνιος 1984. Η εκφώνηση του θεωρήματος είναι η εξής:
"Σε κάθε ορθογώνιο τρίγωνο, τα ημικύκλια που έχουν διάμετρο τις πλευρές του τριγώνου και δεν έχουν άλλα κοινά σημεία με αυτό,
εφάπτονται σε ένα τετράγωνο που έχει πλευρά την ημιπερίμετρο του τριγώνου".
Ζητείται η απόδειξη του θεωρήματος.

44. Τρίτη 14 Σεπτεμβρίου 2010

Η άσκηση αυτή βασίζεται στην άσκηση που πρότεινε ο Μιχάλης Νάννος "βρείτε τη γωνία χ(42)" και στην προσέγγισή της από τον Σεραφείμ Τσιπέλη.
Να αποδειχθεί η σχέση:
\frac{sin48.sin18}{sin(48+18)} + \frac{sin42.sin54}{sin(42+54)} + \frac{sin12.sin6}{sin(12+6)} = \frac{\sqrt{3}}{2}.

45. Παρασκευή 16 Ιουλίου 2010

Το ερώτημα που θέτω είναι μέρος ενός γενικότερου προβλήματος που τέθηκε στο Φόρουμ και για το οποίο ψάχνω μία πλήρη λύση.
Το ερώτημα ήταν: "Να κατασκευαστεί ευθεία, που να χωρίζει ένα τετράπλευρο σε δύο ισεμβαδικά και ταυτόχρονα ισοπεριμετρικά σχήματα". Είναι το πρόβλημα viewtopic.php?f=62&t=8093.
Το ερώτημα που θέτω είναι: "Να εξεταστεί αν υπάρχει ευθεία που να χωρίζει ορθογώνιο και σκαληνό τρίγωνο σε δύο σχήματα ισεμβαδικά και ισοπεριμετρικά".

46. Πέμπτη 1 Ιουλίου 2010

Στο τετράγωνο ΑΒΓΔ φέρουμε τις διαγώνιες ΑΓ και ΒΔ που τέμνονται στο Ο, και χαράσουμε το τεταρτοκύκλιο (εντός του τετραγώνου) με κέντρο Δ και ακτίνα ΔΑ. Αυτό τέμνει την ΒΔ στο σημείο Ε.
Να υπολογιστεί η ακτίνα του κύκλου που εφάπτεται στο ΑΟ, στο ΟΕ και στο τόξο ΑΕ, συναρτήσει της πλευράς α του ΑΒΓΔ.

47. Πέμπτη 15 Απριλίου 2010

Δίνονται τα σημεία Α(2α, 0) Β(4α, 0) και Γ(6α, 0) με α > 0 και ένα ημικύκλιο με διάμετρο ΑΓ, κέντρο Β και ακτίνα 2α.
Φέρουμε την εφαπτομένη ευθεία ε από το (0, 0) προς το ημικύκλιο.
Προβάλλουμε κάθε σημείο Σ του ημικυκλίου στον άξονα xx' και στην ευθεία ε και ονομάζουμε τις προβολές του, Ε και Ζ αντίστοιχα.
1. Να βρεθεί εκείνο το σημείο Σ, ώστε το εμβαδόν του τετραπλεύρου ΟΕΣΖ να είναι το μέγιστο.
2. Να εξετάσετε αν υπάρχει σημείο Σ του ημικυκλίου, ώστε το εμβαδόν του τετραπλεύρου ΟΕΣΖ να ισούται με αυτό του ημικυκλίου.

48. Τρίτη 13 Απριλίου 2010

Δίνεται η συνάρτηση f με f(x) = (x^2)*ln(x) για x > 0 και f(0) = 0.
Να αποδειχθεί ότι το εμβαδόν μεταξύ της γραφικής παράστασης της f και της ευθείας y = x είναι μικρότερο του 2.

49. Δευτέρα 22 Μαρτίου 2010
Είναι μία άσκηση βασισμένη σε αυτήν που πρότεινε ο math-finder.
Δίνεται τετράγωνο ΑΒΓΔ με μήκος πλευράς α και τετράγωνο ΕΖΗΘ με μήκος πλευράς β, με α = 3β,  ώστε το δεύτερο να βρίσκεται στο εσωτερικό του πρώτου.
Να βρεθεί η θέση του ΕΖΗΘ, ώστε το άθροισμα των αποστάσεων των κορυφών του από τις πλευρές του ΑΒΓΔ να είναι το ελάχιστο δυνατό.
Η άσκηση μπορεί να γενικευτεί για α = νβ.
Η λύση της δίνει και άλλη ενδιαφέρουσα γενίκευση.

50. Πέμπτη 18 Μαρτίου 2010

Να αποδειχθεί ότι δεν υπάρχει σημείο μιας έλλειψης, από το οποίο ο μεγάλος ή ο μικρός της άξονας να "φαίνεται" υπό ορθή γωνία.

51. Κυριακή 7 Μαρτίου 2010

Δίνεται ισόπλευρο τρίγωνο ΑΒΓ και ο εγγεγραμμένος σε αυτό κύκλος, ο οποίος εφάπτεται στις πλευρές ΑΒ, ΑΓ, ΒΓ στα σημεία Δ, Ε, Ζ αντίστοιχα.
Να βρεθεί σημείο Σ της ΒΓ, ώστε η ευθεία ΑΣ να αποκόπτει από τον κύκλο, κυκλικό τμήμα ισεμβαδικό με αυτό του μικτόγραμμου χωρίου ΑΔΕ.

52. Δευτέρα 8 Μαρτίου 2010

Δίνονται δύο ίσα ισόπλευρα τρίγωνα.
Στο πρώτο σχεδιάζουμε ένα τετράγωνο που όλες οι κορυφές του είναι σημεία του τριγώνου.
Στο δεύτερο σχεδιάζουμε ένα τετράγωνο που μόνο οι τρεις κορυφές του είναι σημεία του τριγώνου.
Αιτιολογήστε ποιο από τα δύο τετράγωνα έχει μεγαλύτερο εμβαδόν.
 

 53. Σάββατο 27 Φεβρουαρίου 2010

Να αποδειχθεί η ανισότητα:
x(a-z) + y(a-x) + z(a-y) < a^2, αν a >0 και x, y, z αριθμοί του διαστήματος [0, a].
Η άσκηση τέθηκε στο τελευταίο τεύχος του Ευκλείδη Β, σελίδα 48 από τον Γιώργο Τσαπακίδη και εκεί αποδεικνύεται με γεωμετρικό (άκρως ενδιαφέροντα και ελκυστικό) τρόπο.
Εδώ επιπλέον θέτουμε το ερώτημα, αν και πότε ισχύει η ισότητα στη ζητούμενη σχέση.

54. Πέμπτη 25 Φεβρουαρίου 2010

Δίνεται τετράγωνο ΑΒΓΔ. Να εξεταστεί αν υπάρχει ισόπλευρο τρίγωνο με τις κορυφές του επί των πλευρών του ΑΒΓΔ, όταν καμιά από αυτές δεν συμπίπτει με κορυφές του τετραγώνου.
ΣΗΜΕΙΩΣΗ: Η περίπτωση "εγγεγραμμένου" ισόπλευρου τριγώνου σε τετράγωνο, με τον σχετικό υπολογισμό του μήκους της πλευράς του, δόθηκε από τον Γιώργο Τσαπακίδη στο τελευταίο τεύχος του "Ευκλείδη Β", σ. 45.

55.  Κυριακή 28 Φεβρουαρίου 2010

Δίνεται ισόπλευρο τρίγωνο πλευράς α. Να εξεταστεί αν υπάρχει σημείο στο εσωτερικό του τρίγωνου αυτού (εκτός από το κέντρο βάρους του), τέτοιο ώστε το άθροισμα των αποστάσεων του από τις κορυφές του τριγώνου, να ισούται με α\sqrt{3}.
Περιορισμός: Να μην γίνει χρήση θεωρημάτων του Διαφορικού Λογισμού.

56. Πέμπτη 11 Φεβρουαρίου 2010

Να υπολογιστεί το παρακάτω όριο.
\displaystyle \lim_{x\rightarrow 0^{+}} \left(\ln(x)+\frac{1}{x}\right)

57. Παρασκευή 15 Απριλίου 2011

Δίνονται οι ευθείες: y = 0, y = 4, y = 2x, y = 2x - 8.
Πόσες ελλείψεις υπάρχουν, τέτοιες ώστε κάθε μία από τις ευθείες αυτές
να έχει ένα μόνο κοινό σημείο με την έλλειψη.
Προφανώς, το ερώτημα μπορεί να τεθεί για κάθε παραλληλόγραμμο.

58. Παρασκευή 15 Απριλίου 2011

Δίνεται η συνάρτηση f(x)= \frac{lnx}{x} με x > 0.

Να εξεταστεί αν υπάρχει εφαπτομένη ευθεία στη γραφική παράσταση της συνάρτησης f,
η οποία να την τέμνει στο σημείο (3/2, f(3/2)).

59. Κυριακή 19 Απριλίου 2011

Το θέμα που προτείνω είναι πάλι από διαγωνισμό Πανρωσικό.
Αυτή τη φορά κοίταξα καλά όλα τα σχετικά θέματα που έχουν
προταθεί στη στήλη "Γεωμετρία διαγωνισμών για Seniors".
Αλλά ποτέ δεν πρέπει να είναι κανείς σίγουρος.
Το ζητούμενο πάλι είναι να δοθούν πολλές και διαφορετικές λύσεις.
Το θέμα είχε χωριστεί σε δύο σκέλη α) και β) όπως το γράφω.

α) Κάθε μία από τις διαγώνιες ενός κυρτού τετραπλεύρου το διαιρεί σε δύο ισεμβαδικά σχήματα. Να αποδείξετε ότι το τετράπλευρο είναι παραλληλόγραμμο.
β) Δίνεται κυρτό εξάγωνο ΑΒΓΔΕΖ. Αν κάθε μία από τις διαγώνιες ΑΔ, ΒΕ και ΓΖ το διαιρεί σε δύο ισεμβαδικά σχήματα, να αποδείξετε ότι οι τρεις αυτές διαγώνιοι έχουν ένα κοινό σημείο.

60. Τετάρτη 20 Απριλίου 2011
Δίνεται ένας κύκλος (Ο, ρ) και ένας κύκλος (Κ, α) με 5α = ρ, που εφάπτεται εξωτερικά στον (Ο, ρ).
Θεωρούμε ένα σημείο Ν του μικρού κύκλου, φέρουμε το τμήμα ΟΝ
και έστω Χ το μέσο του. Αν ο κύκλος (Κ, α) στρέφεται περί τον κύκλο (Ο, ρ), να βρεθεί ο γεωμετρικός τόπος των σημείων Χ.

61. Πέμπτη 13 Ιανουαρίου 2011
Δίνονται οι κύκλοι (Κ, 4) και (Λ, 2) με ΚΛ = 6.
Να αποδειχθεί ότι υπάρχει τετράγωνο που εφάπτεται στον (Κ, 4) σε δύο ακριβώς σημεία
και στον (Λ, 2) επίσης σε δύο ακριβώς σημεία του.
Να βρεθεί το εμβαδόν αυτού του τετραγώνου.
62. Πέμπτη 21 Απριλίου 2011.
Δίνεται ένας κύκλος (Ο, ρ) και ένας κύκλος (Κ, α) με 5α = ρ,
ο οποίος εφάπτεται εξωτερικά στον (Ο, ρ).
Θεωρούμε ένα σημείο Ν του μικρού κύκλου, φέρουμε το τμήμα ΟΝ και έστω Χ το μέσο του.
Αν ο κύκλος (Κ, α) στρέφεται περί τον κύκλο (Ο, ρ),
να βρεθεί ο γεωμετρικός τόπος των σημείων Χ.
Δεν ήμουν σαφής. Ζητάω την εξίσωση της καμπύλης πάνω στην οποία κινείται το σημείο.
 Με μια ματιά στο σχήμα που έχω αναρτήσει με το πρόγραμμα Cabri II,
φαίνεται ότι το σημείο κινείται σε κλειστή καμπύλη που μοιάζει με μαργαρίτα.
Εξάλλου, το θέμα υποτίθεται είναι για διδάσκοντες.


63. Πέμπτη 13 Οκτωβρίου 2011.

Δίνεται τετράγωνο ΑΒΓΔ με ΑΒ = α. Αν Ο το μέσο του ΑΒ, ονομάζουμε Ε
το σημείο τομής του κύκλου (Ο, ΟΑ) και του τμήματος ΔΟ.
Ονομάζουμε Ζ το σημείο τομής του κύκλου (Ο, ΟΑ) με τη διαγώνιο ΑΓ
και Η το σημείο τομής της ΑΓ με το ΟΔ.
1. Να υπολογιστεί το μήκος του ΕΖ συναρτήσει του α.
2. Να υπολογιστεί το εμβαδόν του τριγώνου ΕΖΗ συναρτήσει του α.

64. Σάββατο 15 Οκτωβρίου 2011.

Το ABC είναι ισόπλευρο τρίγωνο.
Ονομάζουμε M, N, τα μέσα των ημικυκλίων με διαμέτρους τις πλευρές AB, AC αντίστοιχα,
τα οποία δεν έχουν άλλα κοινά σημεία με το τριγωνικό χωρίο εκτός από τις κορυφές του.
Να εκφράσετε το μήκος MN συναρτήσει της πλευράς του ισοπλεύρου τριγώνου.
Παραγγελία: Ο τρόπος υπολογισμού να γίνει:
1. Με όμοια τρίγωνα.
2. Με Τριγωνομετρία.
3. Με Αναλυτική Γεωμετρία (αν και δεν ανήκει στον φάκελο Ευκλείδεια Γεωμετρία), είναι παραγγελία για τον Γ. Ρίζο και τους άλλους αιρετικούς, μικρούς και μεγάλους.
4. Με τη χρήση της στροφής ως προς σημείο, (και αυτό αιρετικό, αφού ανήκει στους γεωμετρικούς μετασχηματισμούς - αντιαρχαιοελληνικόν, αλλά ολίγοτερον καταραμένο σε σχέση την αίρεση του Καρτεσίου).
5. Με όποιον άλλο τρόπο μπορείτε, αρκεί να είναι σεμνός και ηθικός.

65. Πέμπτη 15 Δεκεμβρίου 2011.

Δίνεται το ισοσκελές τρίγωνο ABC με AB = AC = 2a και BC = a > 0.
Το σημείο D κινείται στο μείζον τόξο BC του περιγεγραμμένου κύκλου του τριγώνου ABC.
Αν E είναι το σημείο τομής της διχοτόμου της γωνίας BAD με την BD,
να βρεθεί ο γεωμετρικός τόπος του σημείου E συναρτήσει του αριθμού a.

66. Κυριακή, 17 Απριλίου 2011.

http://www.mathematica.gr/forum/viewtopic.php?f=112&t=14876
Το παρακάτω θέμα είναι γνωστό. Δεν αναφέρω την πηγή του, ούτε πότε τέθηκε ως διαγωνιστικό πρόβλημα. 
Το ζητούμενο είναι να δοθεί απάντηση με όσο περισσότερους τρόπους μπορούμε.
Για το λόγο αυτό το έθεσα ως πρόβλημα των "μεγάλων", 
για υπάρχει μία ποικιλία τεχνικών και μεθόδων στην αντιμετώπισή του.
Εκτός αν έχει ξανατεθεί και δεν το ξέρω, οπότε ... 
τα λαγωνικά του Φόρουμ θα μας παραπέμψουν στο ταμείο για τα περαιτέρω.
Από το μέσο Μ της βάσης ΑΓ ενός ισοσκελούς τριγώνου ΑΒΓ 
φέρουμε την κάθετο ΜΗ στην πλευρά ΒΓ.
Το σημείο Ρ είναι το μέσο του τμήματος ΜΗ. 
Να αποδείξετε ότι ΑΗ είναι κάθετο στο ΒΡ. 


67. Σάββατο 15 Οκτωβρίου 2011
Σε μία ευθεία είναι γνωστές οι θέσεις των σημείων Α, Β και C για τα οποία ισχύουν τα εξής: Από τα σημεία A και B διέρχεται ένας κύκλος (c) και το σημείο C ανήκει στην εφαπτομένη του (c)Να βρεθεί ο γεωμετρικός τόπος των σημείων επαφής των εφαπτόμενων του (c) που διέρχονται από το C.

68.Κυριακή 29 Ιανουαρίου 2012.
Δίνεται η έλλειψη με εξίσωση \frac{x^{2}}{a^{2}}+\frac{y^{2}}{b^{2}}=1 με a > b > 0.1. Να αποδειχθεί ότι υπάρχουν μόνο δύο κύκλοι, οι οποίοι εφάπτονται στους άξονες xx' και yy'
έχουν κέντρα με θετικές συντεταγμένες και έχουν μόνο ένα κοινό σημείο με την έλλειψη.
2. Να εκφράσετε την απόσταση των κέντρων αυτών των δύο κύκλων συναρτήσει των αριθμών a, b.
Ακολουθεί σύντομα και η συνέχεια του "σήριαλ". Το φινάλε είναι γνωστό.... Ένας από τους δύο κύκλους θα παντρευτεί την έλλειψη.

Τώρα, ακολουθεί το δεύτερο μέρος του σήριαλ, όπως έχω υποσχεθεί.
Όταν το κέντρο της έλλειψης κινείται στον άξονα xx' και ο ένας άξονας συμμετρίας της είναι παράλληλος με αυτόν,
να βρεθεί ο γεωμετρικός τόπος του "μικρού" κύκλου που "εφάπτεται" στους άξονες και στην έλλειψη.

69. Τετάρτη, 15 Φεβρουαρίου 2012.
Αν μία συνάρτηση με πεδίο ορισμού το R παίρνει μόνο θετικές τιμές στο R και είναι δύο φορές παραγωγίσιμη, τότε δεν μπορεί να είναι κοίλη σε όλο το R.

70. Κυριακή, 8 Ιανουαρίου 2012.
Οι πλευρές κάθε κανονικού εξαγώνου χρωματίζονται μόνο με τα χρώματα γαλάζιοκόκκινο και μαύρο, χωρίς περιορισμό στην επανάληψη των χρωμάτων.
1. Με πόσους διαφορετικούς τρόπους μπορεί να χρωματιστεί ένα κανονικό εξάγωνο;
2. Με πόσους διαφορετικούς τρόπους μπορεί να χρωματιστεί ένα σχήμα που αποτελείται από 3 ίσα εξάγωνα, όπως αυτό που φαίνεται παρακάτω.
3 εξάγωνα.png

71. Πέμπτη, 29 Δεκεμβρίου 2011.
Το ορθογώνιο τρίγωνο ABC έχει AB = 1 , AC = 2 και υποτείνουσα την BC.
Το ορθογώνιο τρίγωνο BCD έχει CD = 3 και υποτείνουσα την BD.
Το ορθογώνιο τρίγωνο BDE έχει υποτείνουσα την BE, η οποία βρίσκεται στην προέκταση της AB όπως στο συνημμένο σχήμα. Να υπολογιστούν τα μήκη BE και DE.
Προφανώς, αν βρεθεί ένας τρόπος επίλυσης, αυτός βοηθά στην απάντηση και για γενικευμένα μήκη, π.χ. AB = aAC = b, κλπ.

72. Τετάρτη, 22 Δεκεμβρίου 2011.
Με αφορμή τη συζήτηση που αναπτύχθηκε εδώ:
http://www.mathematica.gr/forum/viewtopic.php?f=112&t=20919

Να αποδειχθεί με τη βοήθεια των ιδιοτήτων των συνεχών συναρτήσεων ότι το πρόβλημα που έθεσε ο καλός φίλος Νίκος Κυριαζής έχει λύση.
Το πρόβλημα είναι το εξής: 
Δίνεται μία ευθεία (a) και τα διατεταγμένα σημεία της A, C, B με AC < CB
Να αποδειχθεί ότι υπάρχει σημείο D της (a) τέτοιο ώστε να ισχύει:
\frac{AC^{2}}{CB^{2}}= \frac{AD}{DB}.

73. Πέμπτη, 5 Απριλίου 2012.
Το θέμα που παρουσιάζουμε είναι μία μερική γενίκευση του θέματος που προτάθηκε εδώ:
http://www.mathematica.gr/forum/viewtopic.php?f=20&t=24633
Το σημείο G κινείται στην διαγώνιο AD του κανονικού εξαγώνου ABCDEF.
Αν το FGHJKL είναι και αυτό κανονικό εξαγώνο, να αποδειχθεί ότι η κορυφή J κινείται στον φορέα της πλευράς CD.
Επίσης, να αποδειχθεί ότι ο περιγεγραμμένος κύκλος του FGHJKL διέρχεται και από το σημείο D.
Ερώτηση: Πώς μπορεί να γενικευθούν τα δύο παραπάνω προβλήματα που αφορούν το τετράγωνο και το κανονικό εξάγωνο.
Αν υπάρχει γενίκευση μπορούμε να την αποκαλέσουμε θεώρημα του κινούμενου σημείου στη διαγώνιο κανονικού πολυγώνου;
Σημείωση: Δεν πρόσεξα ότι το έθεσα ως θέμα για την Β Λυκείου. Νομίζω ότι είναι κατάλληλο και για επίπεδο γνώσεων Α τάξης του Λυκείου.
εξάγωνο κινείται σε εξάγωνο.png
Σύμφωνα με όσα γράφτηκαν στη συνέχεια για το θέμα:
http://www.mathematica.gr/forum/viewtopic.php?f=20&t=24633
το προτεινόμενο θέμα επεκτείνεται ως εξής:
Να αποδειχθεί επίσης, ότι η κορυφή του πολυγώνου - η διαδοχική αυτής που κινείται σε ευθεία - κινείται και αυτή σε ευθεία γραμμή.
Στο εξάγωνο είναι η κορυφή Κ.
Τίθεται τώρα το ερώτημα: Σε τι γεωμετρικούς τόπους κινούνται οι άλλες κορυφές του κανονικού πολυγώνου;
Προσωπικά, θεωρώ το θέμα ελκυστικό.

74. Παρασκευή, 1 Μαρτίου, 2013.
Ένα δεδομένο τετράγωνο να χωριστεί σε μικρότερα τετράγωνα έτσι ώστε:
Να χωρίζεται σε 4 τετράγωνα.
Να χωρίζεται σε τετράγωνα.
Να χωρίζεται σε 6 τετράγωνα.
....................................
Να χωρίζεται σε 20 τετράγωνα.

Να στείλετε τις λύσεις με μορφή σχήματος για κάθε μία από τις 17 περιπτώσεις.
Σας πληροφορώ ότι μία ομάδα μαθητών του Δημοτικού του Σχολείου μου (Π.Π.Σ.Π.Θ.) (4η τάξη) 
σήμερα στο μάθημα βρήκε πολλά από αυτά τα ζητούμενα.
Υπόδειξη: Το πρόβλημα δεν αναφέρει ότι τα τετράγωνα πρέπει να είναι ίσα μεταξύ τους.
20 τετραγωνα.JPG8 τετραγωνα.JPG

75. Κυριακή, 28 Απριλίου 2013.
Δίνεται το τρίγωνο ABC όπως στο συνημμένο σχήμα και σημείο F στο εσωτερικό του. 
Τα ευθύγραμμα τμήματα BD και CE τέμνονται στο σημείο F.
Αν το εμβαδόν του τριγώνου BEF ισούται με a
το εμβαδόν του τριγώνου BFC ισούται με b
το εμβαδόν του τριγώνου DFC ισούται με c
να εκφραστεί το εμβαδόν του τετραπλεύρου AEFD συναρτήσει των ποσοτήτων abc.
(Την βρήκα στο Διαδίκτυο, δεν ξέρω αν έχει προταθεί στο Φόρουμ).
εμβαδόν τετραπλεύρου.png
76.Δευτέρα, 22 Απριλίου 2013.
Προτείνω ένα γνωστό, με την έννοια ότι υπάρχει σε αρκετά βιβλία, πρόβλημα ελαχιστοποίησης.Αν AB είναι μία χορδή κύκλου και P ένα σημείο στο τόξο AB, 
(δεν έχει σημασία αν το P βρίσκεται στο μικρότερο ή στο μεγαλύτερο τόξο), 
τότε για είναι το άθροισμα AP + PB ελάχιστο, πρέπει το σημείο P να είναι το μέσο του τόξου AB.

vvvv.ggb.png

77. Παρασκευή, 26 Απριλίου 2013.
Για το κυρτό τετράπλευρο ABCD γνωρίζουμε τα εξής:
AB = BC , γωνία ABD = 5 , γωνία DBC = 35 , γωνία CAD = 80. (Το μέτρο όλων των γωνιών είναι σε μοίρες) 
Να βρεθούν οι γωνίες C και D του τετραπλεύρου.

78.Κυριακή, 21 Απριλίου 2013.
Δίνεται ο συνημμένος πίνακας αριθμών. Ζητάμε να απαντηθούν οι ακόλουθες 5 ερωτήσεις.
Προφανώς, είναι ευπρόσδεκτα και νέα ερωτήματα.
Τέτοιου είδους δραστηριότητες, επειδή περιέχουν από πολύ απλά έως πολύ δύσκολα (ως προς την απάντησή τους) ερωτήματα, 
θεωρούμε ότι είναι κατάλληλες για μαθήματα σε μαθηματικούς ομίλους στους οποίους συμμετέχει ένας αριθμός μαθητών 
με διαφορετικές δυνατότητες, γνώσεις, εμπειρία και τεχνική κατάρτιση στην επίλυση προβλημάτων.
Τα ερωτήματα:
1. Πόσους αριθμούς έχει η 100η γραμμή;
2. Ο αριθμός 100 σε ποια γραμμή ανήκει;
3. Ποιος είναι ο ελάχιστος αριθμός γραμμής στην οποία το ψηφίο 0 εμφανίζεται ακριβώς 10 φορές;
4. Ποιος είναι ο ελάχιστος αριθμός γραμμής στην οποία το άθροισμα των αριθμών της είναι μεταξύ 1500 και 1600;
5. Ποιος είναι ο ελάχιστος αριθμός γραμμής στην οποία συναντάμε μόνο τετραψήφιους αριθμούς;



Αριθμός γραμμής
Στήλη 1η
Στήλη 2η
Στήλη 3η
Στήλη 4η
Στήλη 5η
Στήλη 6η
1η
1





2η
2
3




3η
4
5
6



4η
7
8
9
10


5η
11
12
13
14
15

6η
16
17
18
19
20
21
7η






                            
79. Κυριακή, 14 Απριλίου 2013.
Χθες το Σάββατο στο "Καλαμαρί" στη Θεσσαλονίκη σε μία ενδιαφέρουσα ημερίδα για τις Πανελλήνιες Εξετάσεις 
ανάμεσα στα άλλα εμφανίστηκε η συνάρτηση f(x)= \frac{lnx - x}{e^{x}}
για την οποία τονίστηκε η απόδειξη ότι είναι κοίλη έχει αρκετές δυσκολίες. 
Το βράδυ στην ταβέρνα ο Νίκος Ζανταρίδης έδωσε μία απόδειξη, η οποία πράγματι σε ένα σημείο της είχε ένα τέχνασμα. 
Ζητείται μία απόδειξη ότι η δεδομένη συνάρτηση είναι κοίλη.
Ας ασχοληθούν με αυτήν πρώτα οι υποψήφιοι μαθητές για τις εξετάσεις και μετά οι υπόλοιποι.


80. Τετάρτη, 10 Απριλίου 2013.
Να λυθεί η παρακάτω εξίσωση με δεδομένο ότι xyz είναι φυσικοί αριθμοί.  x^{4} = 10^{y}+4^{2z+1}.







______________________________________________________________

Προβλήματα από το διάσημο
Ουγγρικό μαθηματικό περιοδικό Komal
των ετών 1997 και 1998.

September 1997

C. 473. Can it happen for a whole calendar year that no single Sunday falls on the seventh day of a month?
C. 474. A passanger has been walking for 3.5 hours, covering exactly 5 km in the course of any period of one hour. Is it possible that his average speed during his walk exceeded 5 km/h?
C. 475. In a triangle ABC, a point P on median CC1 is selected such that CP/PC1=m/n. Find the ratios in which P divides the segments of lines AP resp. BP lying inside the triangle.
C. 476. In a right circular cone, both the diameter of the base and the slant height are 20 cm. Find the maximum length of a 2 cm wide self-adhesive band that can be sticked on the lateral surface of the cone without any creasing, cutting, or overlapping.
Gy. 3142. Find the smallest positive integer that is divisible by 28, ends in 28 (in decimal system), and the sum of whose digits is 28.
Gy. 3143. Draw a chessboard on the plane. Let A1, A2, ..., A32 and B1, B2, ..., B32 denote the midpoints of the white and the black fields, respectively. Let furthermore P be any point of the chessboard. Prove that   A1P2+ A2P2+ A32P2= B1P2+ B2P2+ B32P2.
Gy. 3144. Decide if 7/17 can be expressed as 1/a+1/b, where a and b are positive integers.
Gy. 3145. Solve inequality
Gy. 3146. Let a and b denote the lengths of the legs of a right triangle, and r its inradius. Prove that

Gy. 3147. Let P be a point inside a regular hexagon, and e a line incident to P and parallel to a side of the hexagon. Draw five additional lines passing through P such that among the six lines, any two consecutive lines form an angle of 30o. These lines divide the hexagon into 12 regions. Prove that these 12 regions can be divided into three groups such that the sum of the areas of the regions is the same in each group.
Gy. 3148. Let e1, e1 and e1 be three pairwise skew edges of a cube. Select a point Ei on each edge ei for i=1,2,3. Find the locus of the centroids of the triangles E1E2E3.
Gy. 3149. In a triangle ABC, side AB is of unit length. The angles which include this side measure 15o and 60o, respectively. Find the lengths of the two other sides of the triangle without the help of trigonometric functions.
 F. 3187. Find those triangles which satisfy a2+b2+c2=8R2 where a, b, c and R denote the lengths of the sides and the circumradius of the triangle, respectively.
F. 3188. There are given a circle centered at O, a point A selected on the circle, and a straight line d passing through O. A secant starting at A intersects the circle and line d at points B and D, respectively. Prove that upon rotating the secant about A, in any position, the circle incident to points O, B and D passes through an other fixed point different from O.
N. 144. Let n denote an arbitrary positive integer. Prove that 2.(3n)! is divisible by n!(n+1)!(n+2)!.
N. 145. Does there exist a polynomial f(x,y,z) of real coefficients such that, f(x,y,z) is positive if and only if |x|, |y| and |z| are the sides of a triangle?
N. 147. Is it always true that in a tetrahedron, an inner point of each face can be selected such that they form the vertex set of a regular tetrahedron?

October 1997

C. 477. A game machine accepts two kinds of coins: red and green ones. For each coin the machine returns 5 coins of the other kind. One starts playing with 1 green coin. Can it happen that after a while he has as many green coins as red ones?
C. 478. In an arithmetic progression, the sum of the first n terms and the sum of the first 2n terms are A and B, respectively. Express the sum of the first 3n elements in terms of A and B.
C. 479. The lengths of the bases of a trapezium are a and c, respectively. Find the length of the segment parallel to the bases which halves the area of the trapezium.
C. 480. In a tetrahedron, two of the faces are equilateral triangles of unit side. The two other faces are right isosceles triangles. Find the volume of the tetrahedron.
Gy. 3150. In a certain town, each bus line has 3 stops. Any two bus-stops are connected with a line, and any two lines have a common bus-stop. How many bus lines may be there in the town?
Gy. 3155. Prove that in any triangle, a line passing through the incenter halves the perimeter of the triangle if and only if it halves the area of the triangle.
Gy. 3156. In a triangle, the segments of the medians lying inside the inscribed circle are all of the same length. Does this imply that the triangle is equilateral?
Gy. 3157. In a tetrahedron, the segments of the medians lying inside the inscribed sphere are all of the same length. Does this imply that the tetrahedron is regular?
F. 3192. Is it possible to move a knight on a 5x5 chessboard so that it returns to its original position after having visited each field of the board exactly once?
F. 3193. May a non-planar quadrilateral have only right angles?
F. 3194. In a right triangle ABC, construct the point D on the hypotenuse AB such that triangles DCA and DCB have equal inradii.
F. 3195. The whole surface of a cube shaped cake (including its bottom) is covered with chocolate. The cake is to be distributed among K people so that each of them receives the same amount of cake, and also the same amount of chocolate cover at the same time. In order to accomplish this task, we divide the cake into NxNxN alike cube shaped pieces, and give everybody the same number of pieces, taking care that the total surface area of the chocolate covered sides of the pieces one gets is also the same for everybody. Is it possible to realize this for an arbitrary K? At least how many pieces are necessary for K=1997?
N. 149. The sequence (an) is defined by the recursion a0=a1=1, (n+1)an+1=(2n+1)an+3nan-1. Prove that the sequence consists of integer numbers.
N. 150. There are given in the plane a parabola and points P and Q outside the parabola such that line PQ passes through the focus of the parabola. Draw two tangents to the parabola from each of points P and Q. Prove that the four points of intersection obtained this way lie on the same circle.

November 1997
C. 481. There is a company sitting at a round-table. They all leave the table for a while, and after returning to the table they all find that their neighbours differ from those they had previously. How many people may be sitting around the table?
C. 482. Prove that if y3x+1<x+y3 holds for some real numbers x and y, then they also satisfy inequality x3y+1<y+x3.
C. 483. The following question was presented at a Jeopardy show. Which plane figure has the property that its area is half the product of its diameters? According to te official answer, this is `the kite'. Are kites the only plane figures with the above property?
C. 484. In a right circular cone, the angle formed by the axis and a generator measures . Consider the ball inscribed in the cone. Find the ratio of the volume of the ball and that of the cone.
Gy. 3160. In an infinite arithmetic progression of distinct positive integers, we replace each term by the sum of its digits. Can it happen that the new sequence is again an arithmetic progression?
Gy. 3161. At a meeting there are n guest, including Mr. Smith. There is also a journalist there who is looking for Mr. Smith. He is aware of the fact that no one at the meeting knows Mr. Smith, who nevertheless knows everybody there. The journalist may approach any guest, point at someone, and ask the guest if s/he knows that specific person.
a) Can the journalist find Mr. Smith for sure with less than n questions?
b) What is the minimum number of questions he has to ask before he can identify Mr. Smith?
Gy. 3162. The lengths of the legs of a right triangle are a and b, respectively. Draw a circle of radius a centered at one endpoint of the hypotenuse, and also a circle of radius b centered at the other endpoint. Prove that the segment of the hypotenuse lying in the intersection of these two discs is as long as the diameter of the incircle of the triangle.
Gy. 3163. Find the minimum number of different lines determined by n non-collinear points in the plane.
Gy. 3164. In a triangle, let S, M, O and K denote respectively the centroid, the orthocentre, the incentre and the circumcentre of the triangle. Suppose that some two of these points coincide. In which of the six possible cases does this imply that the triangle is equilateral?
Gy. 3165. In a teterahedron it can happen that an altitude goes outside the polytope. How many such altitudes may a tetrahedron have?
F. 3196. Define a sequence an by a0 = 1, an+1 = sin an (n = 0, 1, 2, ...). Prove that the sequence nan2 is bounded.
F. 3197. Some pieces are placed on an 8x8 chessboard such that there are exactly 4 pieces in each raw and coloumn of the board. Prove that there can be found 8 pieces among them such that no two of them are in the same row or coloumn.
F. 3200. We are given two segments AB and CD. Fix AB and move CD around, in a position parallel to AB, such that quadrilateral ABCD is a trapezoid in which a circle can be inscribed. Determine the locus of the midpoints of the segments CD.
F. 3201. Let M be an arbitrary interior point of a tetrahedron ABCD. Let N, P, Q and A1, respectively, denote the intersection points of planes BCM, CDM, BDM, and BCD with lines AD, AB, AC and AM. Let finally D1, B1 and C1 denote the points where lines A1N, A1P and A1Q intersect the plane incident to A and parallel to BCD. Prove that A is the centroid of triangle B1C1D1.
ATTENTION! We correct and submit N. 146. again. (See KöMaL 1997/6.):
N. 152. There are given finitely many discs of unit radius in the plane such that the centres of any two of them are at least 10 units apart. Is it true that there exists a (not necessarily closed) polygon whose vertices are the centres of the given discs such that the polygon contains the centre of each disc, and each segment of the polygon intersects only those discs which are centered at its endpoints.

December 1997

C. 485. We are given a 100x100 array of numbers such that, for k=1,2,...,100, its kth row is an arithmetic progression whose first term is 1, and whose common difference is k. Find the largest entry along the diagonal that connects the lower left corner of the array to its upper right corner.
C. 486. How many different ways can some 1, 2, 5, 10 and 20 forint coins make 25 forints?
C. 488. The lateral surface of a frustum of a right circular cone is coloured with two colours; it is blue below the middle of its slant height and is red above that. Find the ratio between the radii of the bases of the frustum, given that the surface area of the blue part is twice as large as that of the red part.
Gy. 3166. Compare the following two numbers: 19971999 19991997
Gy. 3167. Prove that two positive integers a and b have the same parity if and only if there exist positive integers c and d such that a2+b2+c2+1=d2.
Gy. 3168. In a set of size 15, find 15 subsets, each of size 7, such that any two of them have exactly 3 elements in common.
Gy. 3169. Find all triangles with integer side-lengths in which one of the angles is twice as large as an other one.
Gy. 3170. A triangle ABC has a right angle at vertex C. The bisector of the angle at B intersects side AC and the circumcircle at points P and Q, respectively. Find the measures of the angles of the triangle, given that BP = 2PQ.
Gy. 3171. The lengths of the sides of a certain triangle are integers, one of them being 1. Is it possible that the inradius of the triangle is a rational number?
Gy. 3172. A right circular cylinder shaped vessel, whose axis is vertical, is filled up with water. The radius of the cylinder is 10 cm, its height is 25 cm. Find the amount of water which flows out of the vessel when it is tilted by 30o. Construct the angle by which the vessel should be tilted from its original position if we want to pour out half of the water it contains.
Gy. 3173. Given three straight lines in the space, each incident to a given point P, construct a plane through P which forms the same angle with each of the given lines.
F. 3203. Prove that any set of size 15 contains 15 subsets, each of size 6, such that any two of them have either 1 or 3 common elements.
F. 3204. There is given a heap of 923k pebbles. Two players play the following game. They move alternately, removing at each turn either 9, 2 or 3 pebbles from the heap. The one who cannot move, loses the game. Which player has a winning strategy?  
F. 3206. We are given a circle k1, and points H1, H2 and P outside the circle. Construct a circle k2 passing through P such that, for i=1,2, the lengths of the tangents from Hi to k2 are the same as the lengths of the tangents from Hi to k1.
F. 3207. A regular tetrahedron, whose edges are 2 units long, is inscribed into a right circular cylinder in such a way that it has two vertices on the perimeter of each base of the cylinder. Find the volume of the cylinder.
N. 155. In an infinite sequence of positive integers, each term has the same number of divisors. Prove that there is an infinite subsequence in which any two terms have the same greatest common divisor.
N. 157. An nxn array consists of real numbers such that the entries are monotone decreasing in each row and each column of the array. Whenever we are given a number a, our task is to decide if the array contains that number or not. For that purpose, we may compare a with any entry in the array. How many comparisons do we have to make to solve the problem for sure, regardless of the specific value of a?
N. 158. There are given 9 convex subsets of (3-dimensional) space such that any 8 of them have a lattice point in common. Prove that there exists a lattice point which is contained in each set.


January 1998

C. 489. Joe and Charlie went on an excursion. At the end of their hike they arrived at a highway and decided to take a bus. Joe continued to walk forward to the next bus-stop at a speed of 4 km/h, while Charlie assumed that the previous stop was closer by, and thus headed in the opposite direction at a speed of 6 km/h. They each arrived just on time to catch the bus. Find out if Charlie's assumption was right, given that the bus travelled at a speed of 60 km/h.
C. 490. Prove that the difference between any two odd perfect squares is divisible by 8.
C. 491. Prove that every triangle has at most one such side which is smaller than the corresponding altitude.
C. 492. We measure the angles of inclination of a tower, emerging on level ground, at distances of 50 m and 100 m from its base, respectivley. The two measures sum up to 45o. Determine the height of the tower.
Gy. 3174. Ann and Bob play the following game. First they draw an nxn square and colour its horizontal sides with red and its vertical sides with blue. Ann starts the game and then they move alternately. At each move, Ann (Bob) chooses a small square which is not yet coloured and which already has at least one red (blue) side, and colours it with red (blue). The game ends when either Ann (Bob) connects the horizontal (vertical) sides with a chain of red (blue) fields such that any two consecutive ones share a common edge, in which case she (he) wins the game, or when a player, whose turn it is, cannot move, in which case the game is a draw. Determine those values of n for which one of the two players has a winning strategy.
Gy. 3177. There are two Hungarian teams, the Bolyai TC and the E\"otv\"os TK, among those qualified for the 16 best teams in the Europe Cup. How likely is that they are going to play against each other? (After each match, one team qualifies for the next round and the other one is eliminated from the contest.)
Gy. 3178. In a triangle of unit area, what is the smallest possible length of the second largest side?
Gy. 3179. The vertices of tetrahedron BRYG are coloured with blue, red, yellow and green, respectively. Next, the edges of the tetrahedron are coloured such that all the four colours are used and the colour of each edge agrees with the colour of one of its vertices. Prove that there can be found a vertex such that either the three edges starting at that vertex or the other three edges have the three colours blue, red and green, respectively.
Gy. 3181. In a right square based pyramid, the centres of the inscribed and circumscribed spheres coincide. Find the angle formed by any two neighbouring lateral edges.
F. 3208. Given that the positive numbers x, y, z satisfy x2+xy+y2=9, y2+yz+z2=16, z2 + zx + x 2 = 25, determine xy + yz + zx.
F. 3209. Consider a connected graph on n vertices and assign a real number to each edge of the graph which is to be called the value of that edge. For any path of the graph, the value of the path is defined as the largest value of any edge along the given path. For any two vertices x,y of the graph, let f(x,y) denote the smallest possible value of a path connecting x with y. Prove that the cardinality of the range of f is not greater than n-1.  
F. 3212. In a triangle ABC, F denotes the midpoint of side BC and E denotes the common point of BC and the angle bisector starting at A. The circumcircle of triangle AEF intersects sides AB and AC at points B1 and C1, respectively. Show that BB1=CC1.
F. 3213. Consider two skew lines wich are extensions of two edges of a cube, respectively. A unit segment is placed arbitrarily along each line. In which position of the two segments has the tetrahedron determined by their endpoints a maximum volume?
N. 159. Prove that every convex n-gon can be dissected, with nn-3 pairwise non-crossing diagonals, into triangles such that no vertex of the polygon is inside the circumcircle of any triangle.
N. 161. The coefficients of a polynomial p are integers whose absolute values are not greater than 1998. Given that p(2000) is a prime number, prove that p cannot be written as a product of two polynomials, each of positive degree and of integer coefficients.
N. 162. The complete graph G on n vertices is to be decomposed as a union of complete bipartite graphs such that each edge of G belongs to exactly one of the bipartite graphs. Find the minimum number of bipartite graphs needed for such a decomposition.
February 1998
C. 493. Find all perfect squares with the following property: if we divide them by 11, the (partial) quotient is a prime number and the remainder is 4.
C. 494. Write the numbers 1,2,...,n in a row, in increasing order. In the next row, write the same numbers in decreasing order. Consider the difference of any two numbers, one of which is lying below the other. Determine the sum of the absolute values of all such differences.
C. 495. Prove that if the base angles of a trapezoid are not congruent, then the diagonal starting at the vertex of the smaller angle is longer than the other diagonal.
C. 496. The diagonals of a regular hexagonal prism have lengths 12 and 13. Calculate the volume of the polyhedron.
Gy. 3183. Some people, who know each other very well, are sitting at a round table. Some of them always tell the truth, but the others always lie. Each of them claims that he is veracious, but the kth person sitting on his right hand side is a liar. How many people can be there around the table?
Gy. 3184. In America, temperature is measured in Fahrenheit degrees. This is a linear scale in which the melting point of ice is taken as 32 degrees (32 oF) and the boiling point of water is taken as 212 oF. Someone gives us the temperature in Fahrenheit degrees, rounded to the nearest integer degree. We then convert it to the Celsius scale and round it to the nearest integer degree. Express, in centigrades, the maximum possible deviation of the temperature calculated this way from the actual temperature.
Gy. 3185. The numbers 1,2,...,n are assigned, in some order, to the vertices A1, A2, ..., An of a regular n-gon.
a) Prove that the sum of the absolute values of the differences between adjacent numbers is at least 2n-2.
b) Find the number of arrangements in which the above sum is exactly 2n-2.
Gy. 3186. Construct a triangle, given the lengths of an altitude and a median, starting at the same vertex of the triangle, and the distance between that vertex and the orthocentre of the triangle.
Gy. 3187. Ann and Bob agreed to have a date somewhen between 5 and 5:30 p.m. Assuming that they arrive within the specified interval, find the probability that no one has to wait more than 10 minutes for the other.
Gy. 3188. Prove that every tetrahedron has a vertex such that the edges starting at that vertex can be rearranged to form a triangle.
Gy. 3189. Rotate a cube of unit edge by 60o about one of its diagonals. Calculate the volume of the intersection of the rotated cube with the original one.
F. 3214. Aladdin walked all over the equator in such a way that in each moment he either was moving to the west or was moving to the east or applied some magic trick to get to the opposite point of the Earth. We know that he travelled at most 19000 km's alltogether during his westward moves. Prove that there was a moment when the difference between the distances he had covered moving to the east and moving to the west, respectively, was at least half of the length of the equator.
F. 3217. Let s1, s2, s3 denote the lengths of the medians in a triangle, and let d1, d2 and d3 denote the distances of a point P from the medians, respectively. Prove that one of the products s1d1, s2d2 s3d3 equals the sum of the other two.
F. 3218. We are given a triangle A1A2A3 and a point P inside the triangle. Let Bi, for i=1,2,3, denote the foot of the perpendicular from P to line AiAi+1 (indices are taken modulo 3). In a similar manner, we can obtain a triangle C1C2C3 from triangle B1B2B3, and then triangle D1D2D3 from triangle C1C2C3. Prove that triangle D1D2D3 is similar to triangle A1A2A3.
F. 3219. Each face of a regular dodecahedron is coloured with one of four colours, red, blue, yellow and green, such that adjacent faces have different colours. Find the number of such edges for which one of the two faces incident to the edge is blue and the other one is green.
N. 164. Is there any number in the Fibonacci sequence whose six last digits (in decimal system) are all 9?
N. 166. We are given four points on a line in the following order: A,B,C,D. Moreover, we know that AB=CD. Is it possible to construct the midpoint of segment BC, if we are only allowed to use a straightedge?

March 1998
C. 497. Is there any positive integer n such that 1.2.3.....(n-1).n, that is, n!, ends with exactly 100 zeros?
C. 498. Solve inequation x3+1>x2+x.
C. 499. There is given a circle of unit radius in the plane. A wreath of n congruent circles is formed around the given circle such that each circle in the wreath touches its two neighbours and also the given circle. Determine, in terms of n, the common radius of the circles. Calculate these radii, up to four digits precision after the decimal point, for the first four possible values of n.
C. 500. Is it possible to slip two fine arts albums, each 22 cm wide, and a 25 cm wide cookbook into 28 cm wide plastic bag? Assume that each book has a thickness of 1.5 cm.
Gy. 3190. We define a number 44...48...89 with the following procedure. First, we insert the number 48 between the two digits of 49. Next, 48 is inserted between the digits 4 and 8 of the resulting number. Finally, we repeat this second step a few times. Is it true that every number obtained this way is a perfect square?
Gy. 3191. Is it true that every integer has at least as many positive divisors of the form 4k+1 as that of the form 4k-1?
Gy. 3192. Find all pairs of positive integers A,B with the following property: if the decimal representation of A is placed in front of that of B, then the resulting number is a perfect square, equal to twice the product of A and B.
Gy. 3193. A basketful of peanuts is distributed into n piles. We gather the piles and rearrange them into n+k new piles (k>0). Prove that at least k+1 peanuts are transferred to smaller piles than the respective original piles that contained them. (H)
Gy. 3194. In a triangle, two of the altitudes are at least as long as the corresponding sides of the triangle. Find the angles of the triangle.
Gy. 3195. In a right triangle, parallel lines to each side are drawn through the incentre of the triangle. These lines divide each side of the triangle into three parts. Prove that the length of the middle part of the hypotenuse equals the sum of the lengths of the middle parts arising on the legs of the triangle.
Gy. 3196. There are given rays e0, e1, ..., e6, each starting at the same point A of the plane. Each ray forms an angle of 30o with the one that precedes it in the given order. Let B0 denote the point on ray e0 such that A and B0 are a unit distance apart. Let B1 denote the orthogonal projection of B0 to e1. Similarly, denote by B2 the orthogonal projection of B1 to e2, and so on. Calculate the length of the polygonal path B0B1...B6 and the area of polygon B0B1...B6.
Gy. 3197. Four balls, each of radius r, are placed on a horizontal plane such that each of them touches two others and their centres form a square. A fifth ball of the same radius is placed on the top of these balls, touching all of them. Calculate the distance of the topmost point of the fifth ball from the given plane. (H)
F. 3220. A rectangular field is divided into 100 congruent rectangular plots with the help of lines parallel to the edges of the field. Neighbouring plots share a common edge. Initially, 9 plots of the field are overgrown with weeds. The weeds overgrow any plot whose (at least) two neighbours are already overgown. Is it possible that the whole field becomes overgrown sooner or later?
F. 3221. The numbers 1,2,...,n are assigned, in some order, to the vertices of a regular n-gon A1A2...An.
a) Determine the maximum possible value of the sum of the absolute values of the differences between numbers assigned to consecutive vertices.
b) Find the number of arrangements for which the above sum is maximum.
F. 3222. Given that n is a prime, prove that there is no positive integer x that satisfies equality (1n+2n+...+xn)+(1n+2n+...+(n-1)n)=1n+2n+...+(2n-1)n.
F. 3223. A polygon has exactly n axes of symmetry. How many sides may the polygon have?
F. 3224. In a certain convex polytope, every face has a central symmetry. Prove that at least six of the faces are parallelograms.
F. 3225. Find a set of points in 3-space which intersects every plane in at least one but not more than five points.
N. 167. There are given three unit discs in the plane. Let t1 and t2 denote the areas of the two regions in the plane which the discs cover exactly once resp. exactly twice. Prove that .
N. 170. Given that 15a+6b+4c+8d=0, prove that the equation ax3+bx2+cx+d=0 has a positive root.

April 1998

C. 501. One morning in the school the integers from 1 through a certain number were written on the blackboard. A student carefully erased one of them. There would have been nothing special about it if someone had not noted that the average of the remaining numbers had been . Try to figure out which number was erased.
C. 502. Let us denote the roots of equation x2 - 2bx + b2 c2 = 0 by x1 and x2. Prove that the roots of equation x2-2b(b2+3c2)x+(b2-c2)3=0 are and .
C. 503. Given are two isosceles triangles. The inscribed circles of the triangles touch the legs at their points of trisection closer to the base of the triangle in one case and further from the base of the triangle in the other case. In which case covers the inscribed circle a greater portion of the area of the triangle?
C. 504. The points A, B, C, D, E, F lie in the 3 dimensional space. How are the points positioned if there exists a plane, in the same distance from each of the points, which separates points ABC from points DEF?
Gy. 3198. Do there exist positive integers a, b, c, d such that ab = cd and a + b + c + d is a prime number?
Gy. 3199. Find all integer solutions to equation 1996x + 1998y + 1 = xy.
Gy. 3200. Let H be a 1000-element subset of the set {0, 1, 2, ..., 1998}. Prove that it has two not necessarily distinct elements a and b such that a+b is a power of 2.
Gy. 3201. A 9x9 chessboard is is packed with 1x2 dominoes such that only one corner of the board is left uncovered. Prove that the uncovered field of the board can be transferred to any other corner by shifting the dominoes about on the board.
Gy. 3202. A solid cube of edge length n cm is divided into smaller cubes each with egde length 1 cm. Determine those values of n for which a subset of the small cubes can be arranged to form a cube with edge length 2n cm. The latter cube may be hollow.
Gy. 3203. The lengths of the diagonals of a rhomboid are 2a and 2b, respectively. The sides of the rhomboid together with those tangents of the inscribed circle which are parallel to either diagonal form an octagon. Express the area of the octagon in terms of a and b.
Gy. 3204. The segments AB and CD are parallel. P is an interior point of segment BC, the lines AP and CD intersect in E. For which point P is the sum of the areas of triangles APB and CPE is minimum?
Gy. 3205. Semicircles are drawn externally on each side of the acute triangle ABC. The altitudes drawn from vertices A,B,C intersect the semicircles in points E,F,G, respectively. Prove that the hexagon AGBECF can be folded into a pyramid of base ABC.
F. 3226. In how many different ways can one colour the circles depicted on the figure such that each arm contains exactly two yellow and two blue circles? Two colourings are different if one cannot be obtained from the other using a rotation or a reflection.
F. 3227. Find all values of m for which polynomial x m + ym + zm - (x+y+z)m is divisible by (y+z)(z+x)(x+y).
F. 3228. A bus line connects cities A and B. The buses leave in every 30 minutes. Each bus has a capacity of 50 passengers. Those who do not fit in the bus wait in the waiting hall, and those passengers who do not even fit there leave for B on foot. Between two consecutive buses either 0, 25, 50 or 75 passengers arrive, with probability 1/4, respectively. A new waiting hall is to be built to replace the old one. It is to be built from modules, each of capacity 25. At least how many such modules have to be used in order to ensure that the probability that one has to leave on foot is smaller than 1%?
F. 3229. In a triangle of side lengths a,b,c, the exradius belonging to side c is the geometric mean of the exradii belonging to sides a and b, respectively. Express c in terms of a and b.
F. 3230. Let S denote the centroid of triangle ABC. Erect perpendiculars from each vertex of the triangle to the external and internal bisectors of the angles belonging to the other two vertices. Prove that the sum of the squares of the lengths of the resulting perpendicular segments is 6(SA2+SB2+SC2).
F. 3231. Given are a sphere g and a point P in its interior. Let h1, h2 és h3 be pairwise orthogonal chords passing through P. Each of the three planes determined by the pairs of chords intersect the sphere in a circle. Prove that the sum of the areas of the three circles is independent of the position of the chords.
N. 171. There are given lines e1, e2, ..., en and a point P in the plane. Project P perpendicularly on the first line. Next, project its image on the second line, and so on. After projecting on the last line, project again on the first line, and continue the process. Prove that the different images obtained by the procedure form a bounded set.
N. 172. An automatic card shuffler is used to shuffle a deck of 2n cards. It can rearrange the deck according to the rule (1, 2, 3, ..., 2n-1, 2n) (2n, 1, 2n-1, 2, ..., n+1, n). Prove that the cards will be in their original order after at most 2n shuffles.  
N. 174. A tour of a tree graph is the following. A figure, moving along the edges of the graph, visits each vertex such that it travels along every edge exactly twice. We say that two figures tour the graph simultaneously if they start at the same time, not necessarily at the same vertex, and then they always move at the same time during their respective tours of the graph. They meet if they move to the same vertex or stay on the same edge of the graph at the same time. Let f(n) denote the largest integer for which there exist a tree on n vertices which f(n) figures can tour simultaneously such that no two of them meet. Prove that f(5k+1)=2k for every positive integer k.

May 1998

C. 505.  Let us call a calendar week even or odd according to the parity of the sum of the numbers of the days in the seven dates making up that week. How many even weeks can there be among any 52 consecutive weeks that counted from the first Monday in January?
C. 506. Find all pairs of integers m and n for which the equation
(2m-3)(n-1)x2+(2m-3)(n-1)(m-n-4)x-2(2m-3)(n-1)(m-n-2)-1=0 has integer solutions.
C. 507. Find the locus of the vertex of the parabola y=x2+tx+1 where t varies through the real numbers.
C. 508. Our train travels on a straight railway track at a constant velocity of 26 m/s. Looking out of the window, we notice a cylindrical grain silo. As we approach the silo for 5 seconds, our distance from the silo is reduced by 100 m, while the silo seems to turn through an angle of 5o. How much longer do we continue to get closer to the silo?
Gy. 3206. Given 2n distinct points on a line; points A1, A2, ..., An are red and points B1, B2, ..., Bn are blue. Consider all segments determined by the given points, and draw circles with these segments as diameters. A circle is then coloured red if the endpoints of its diameter are of the same colour. The other circles are coloured blue. Prove that the total length of the perimeters of the blue circles is at least as large as that of the red circles.
Gy. 3207. A certain number can be written in the form 3a2+32b2, where a and b are natural numbers. Multiply the number by 97. Prove that the new number can be expressed in that form too.
Gy. 3208. Solve the equation x(x-y) + y(y-z) + z(z-x) = 1 in the set of integers.
Gy. 3209. Which polynomials p(x) satisfy the following equality: (x-16)p(2x)=16(x-1)p(x)?
Gy. 3210. An inscribed trapezium is divided by its two diagonals into four triangles. Prove that each triangle has a circumcentre lying on one of the perpendicular lines dropped from the intersection point of the diagonals onto the sides of the trapezium.
Gy. 3211. There are given infinitely many rectangles in a Cartesian coordinate system such that each rectangle has an edge lying on the x axis and another one lying on the y axis. Moreover, the vertices of the rectangles opposite to the origin have integer coordinates. Prove that there are two rectangles such that one is contained in the other.
Gy. 3212. There are given 6 points in the plane, no three collinear. A few pairs of the given points are connected with line segments. At least how many segments must be drawn so that there should always be a triangle formed with its vertices being three of the given points?
Gy. 3213. Given a convex pentagon and a point P on its boundary, construct a line passing through P that halves the area of the pentagon.
F. 3234. In an all-aginst-all tournament, three teams are said to `beat around' each other if each of them wins exactly once when only games among those three teams are considered. At most how many beat-arounds may happen in a tournament with 23 participating teams?
F. 3235. Two 1998-gons are drawn in the plane such that the midpoints of their sides coincide. Does it imply that the polygons have equal areas?
F. 3236. Given a convex polygon K and a point P inside the polygon, prove that there exists a direction such that of all lines parallel to it, the one passing through P has the longest segment in K.
F. 3237. The diagonals of a certain four-sided prism pass through a common point. Prove that the prism is a paralleloepiped.
N. 175. There are n natural numbers written along a circle. We write the absolute value of their difference between every two consecutive numbers, and then erase the original numbers. We may repeat this procedure a finite number of times. Determine those values of n for which all numbers along the circle will be eventually zero, regardless of the particular choice of the original numbers.
N. 177. An automatic card shuffler is designed to shuffle, for any positive integer n, a deck of 2n cards according to the following rule: (1,2,3,...,2n) (2n,1,2n-1,...,n+1,n). We start with 1998 different cards. We complete the deck by adding 2n-1998 new cards to it, and feed the deck of 2n cards, in some order, to the machine. Is it possible to choose n and the order of the cards in such a way that every permutation of the original 1998 cards should occur in some finite number of shuffles?


__________________________________________________________

ΚΑΤΑΣΚΕΥΗ ΜΑΘΗΜΑΤΙΚΩΝ ΠΡΟΒΛΗΜΑΤΩΝ

ΜΙΑ ΔΙΑΔΙΚΑΣΙΑ ΠΟΥ ΕΝΔΙΑΦΕΡΕΙ ΤΗ ΔΙΔΑΚΤΙΚΗ ΤΩΝ ΜΑΘΗΜΑΤΙΚΩΝ

Πούλος Ανδρέας, μαθηματικός

Εισήγηση που παρουσιάστηκε στην 5η Μαθηματική Εβδομάδα  της Ε.Μ.Ε. Μάρτιος 2013

Ο διακεκριμένος προπονητής της Ολυμπιακής Ομάδας των Μαθηματικών της Βρετανίας Tonny Gardner,  ο οποίος έχει γράψει αρκετά βιβλία σχετικά με την επίλυση μαθηματικών προβλημάτων σε ένα κείμενό του (1993, σελίδα 58) αναφέρει το εξής: «Όποτε ανοίγω το περιοδικό Mathematics Competitions ή το τελευταίο τεύχος του Crux ή όταν παραλαμβάνω έντυπο υλικό από διοργανωτές μαθηματικών διαγωνισμών από όλο τον κόσμο, αισθάνομαι ένα βαθύ θαυμασμό για όλους εκείνους που κατασκευάζουν μαθηματικά προβλήματα. Επιπλέον,  όταν επιχειρώ να επιλύσω ορισμένα από αυτά τα προβλήματά τους, αυτός ο θαυμασμός επαυξάνεται από μία αίσθηση έντονης ταπείνωσης». Αυτή η αναφορά από έναν ειδικό στην επίλυση, αλλά και στην κατασκευή μαθηματικών προβλημάτων Ολυμπιακού επιπέδου, τονίζει πόσο σημαντική διαδικασία είναι η κατασκευή μαθηματικών προβλημάτων. Μάλιστα, από τα συμφραζόμενα του Gardner προκύπτει ότι, αυτός τη θεωρεί ανώτερη από την ίδια την επίλυση προβλημάτων. Με πολύ σαφή τρόπο οι A. Einstein και L. Infeld (1938, σελ. 92) τόνισαν τη σημασία της διατύπωσης νέων προβλημάτων για την ανάπτυξη της επιστήμης, «Η διαμόρφωση ενός προβλήματος συχνά είναι σημαντικότερη από τη λύση του, η οποία μπορεί να είναι απλά ένα σύνολο μαθηματικών η πειραματικών τεχνικών. Το να θέτουμε νέα ερωτήματα, νέες πιθανές εκδοχές, να επανεξετάζουμε παλαιά ερωτήματα με νέα οπτική γωνία, απαιτεί δημιουργική φαντασία και συνιστά ουσιώδη πρόοδο στην επιστήμη».
Οι δύο διαδικασίες, η κατασκευή μαθηματικών προβλημάτων, η οποία στην αγγλική γλώσσα περιγράφεται ως problem posing (στο εξής συντομογραφικά θα την αναφέρουμε ως Ρ.Ρ.) και η επίλυση μαθηματικών προβλημάτων, η οποία στην αγγλική γλώσσα περιγράφεται ως problem solving (συντομογραφικά θα την αναφέρουμε ως P.S.) είναι δύο διαδικασίες που η μία βασίζεται στην ύπαρξη της άλλης. Δηλαδή, δεν έχει νόημα η κατασκευή προβλημάτων, αν δεν τεθεί ο στόχος της επίλυσής τους και αντίστροφα δεν μπορούμε να επιλύσουμε ένα πρόβλημα το οποίο δεν έχει διατυπωθεί και δεν έχει τεθεί ως τέτοιο. Για πολλά χρόνια το ενδιαφέρον της Διδακτικής των Μαθηματικών ήταν στραμμένο κυρίως στη διαδικασία P.S., ενώ από τη δεκαετία του 1980 και μετά παρουσιάζονται μελέτες, έρευνες και άρθρα που αφορούν και τη διαδικασία του Ρ.Ρ.  Δεν είναι τυχαίο ότι οι Walter και Brown (2005,  σελ. 4) τονίζουν ότι «Το να θέτουμε ερωτήματα ή η σύνθεση προβλημάτων είναι ένας στόχος πολύ πιο σημαντικός απ’ ότι συνήθως πιστεύουμε».  
Στη χώρα μας είναι σχετικά περιορισμένο το ενδιαφέρον της Διδακτικής των Μαθηματικών στο Ρ.Ρ. Αυτός είναι ένας από τους βασικούς λόγους παρουσίασης αυτής της συγκεκριμένης εργασίας, η οποία εκτός της περιγραφής και παράθεσης δεδομένων, στοχεύει και σε μία απλή σύνθεση, σε συγκρίσεις και συμπεράσματα που προκύπτουν από ένα πλήθος σύγχρονων ερευνών που αφορούν το Ρ.Ρ.
Η εμπειρία δείχνει ότι η διαδικασία του Ρ.Ρ. είναι πιο δύσκολη από αυτή του Ρ.S. Η κατασκευή ενός «καλού» προβλήματος απαιτεί περισσότερο χρόνο από αυτόν που απαιτεί η επίλυσή του. Δεν είναι τυχαίο ότι, όσοι προτείνουν σε άλλους την επίλυση μαθηματικών προβλημάτων επιλέγουν έτοιμα προβλήματα, ακόμα και αν δεν γνωρίζουν τη λύση τους, αρκεί να διαισθάνονται τι είδους γνώσεις και μαθηματικά εργαλεία απαιτούνται γι’ αυτήν. Ακόμα και οι πολύ δυνατοί λύτες γνωρίζουν καλά ότι η κατασκευή ενός προβλήματος με απαιτήσεις, είναι ιδιαίτερα χρονοβόρα και δεν οδηγεί πάντα σε επιθυμητό αποτέλεσμα. Επίσης, τους φαίνεται περισσότερο οικεία η προσπάθεια επίλυσης ενός προβλήματος, από αυτή της κατασκευής του. Δεν αναφερόμαστε σε προβλήματα που έχουν πηγάσει από την ίδια τη μαθηματική έρευνα, τα οποία είναι ένας μεγάλος αριθμός προβλημάτων, αλλά για προβλήματα που συνθέτουμε για έναν συγκεκριμένο σκοπό, όπως είναι η εξακρίβωση της κατανόησης εκ μέρους του λύτη των μαθηματικών εννοιών που αυτά εμπεριέχουν, της κατανόησης των σχέσεων μεταξύ των δεδομένων του προβλήματος, της ταχύτητας επίλυσης, της εύρεσης της συντομότερης, της πλέον ευφυούς λύσης και άλλων παραμέτρων, τις οποίες θέτει ο κατασκευαστής. Χαρακτηριστικά παραδείγματα τέτοιων προβλημάτων είναι αυτά των μαθηματικών διαγωνισμών.
Από μία άποψη, αποτελεί προτεραιότητα στη μαθηματική εκπαίδευση να μάθουν οι μαθητές – αλλά και οι διδάσκοντες (αυτός είναι στόχος, ο οποίος θεωρείται αυτονόητος χωρίς να είναι πάντα τέτοιος) να επιλύουν μαθηματικά προβλήματα. Ο στόχος της κατασκευής μαθηματικών προβλημάτων δεν αφορά την πλειοψηφία των μαθητών- φοιτητών-σπουδαστών, αλλά και των δασκάλων τους. Ο λόγος είναι απλός, μία μειοψηφία «ειδικών» έχει αναλάβει τον ρόλο της παραγωγής προβλημάτων για κάθε είδος και για κάθε απαίτηση. Αυτό συμβαίνει στη μαθηματική εκπαίδευση όλων των βαθμίδων και σε όλες τις χώρες του κόσμου. Ας ασχοληθούμε όμως, με τα εκπαιδευτικά και παιδαγωγικά δεδομένα της χώρας μας. Από τις αρχές της νεοελληνικής μαθηματικής εκπαίδευσης, ο μόνος τρόπος για να μάθουν να επιλύουν οι δάσκαλοι και οι μαθητές προβλήματα ήταν αυτός του παραδείγματος και της επίμονης προσπάθειας επίλυσης νέων άγνωστων προβλημάτων. Ας σημειωθεί επίσης, ότι μέχρι την καθιέρωση των εισαγωγικών εξετάσεων στα Πανεπιστήμια, η επίλυση ασκήσεων και προβλημάτων εθεωρείτο πολυτέλεια και κυριαρχούσε η θεωρητικολογία. Πολύ αργότερα, μετά τη δεκαετία του 1980, η μαθηματική μας εκπαίδευση συνειδητοποίησε ότι είχε την ανάγκη σε μαζική κλίμακα από βιβλία που να περιγράφουν, να συστηματοποιούν και να δίνουν ιδέες και κίνητρα για την επίλυση των μαθηματικών προβλημάτων κυρίως σε επίπεδο διδασκόντων. Αυτοί με τη σειρά τους θα προσάρμοζαν αυτό το διδακτικό θεωρητικό υλικό στις ανάγκες και δυνατότητες των μαθητών τους και με τον τρόπο θα βελτίωναν τις τεχνικές και την ποιότητα των μεθόδων επίλυσης προβλημάτων. Μία χαρακτηριστική προσπάθεια ήταν αυτή της δεκαετίας του 1990, όταν το κλασσικό βιβλίο του George Polya που γράφτηκε το 1944 με τον τίτλο "How to solve it (Πώς να το λύσω)", μεταφράστηκε το 1990 από τις εκδόσεις «Σπηλιώτης» και το 1998 από τις εκδόσεις «Καρδαμίτσας» με επιμέλεια του Τάσου Πατρώνη. Στο βιβλίο αυτό, ο Polya αναπτύσσει ένα γενικό σχέδιο για την εφαρμογή μιας επιτυχούς διαδικασίας για την επίλυση μαθηματικών προβλημάτων. Το βιβλίο διαβάστηκε πολύ από τους Έλληνες εκπαιδευτικούς και αποτελεί πηγή αναφοράς σε πολλές έρευνες και άρθρα που αφορούν το P.S. Είναι απαραίτητο να σημειωθεί ότι το How to solve it για πρώτη φορά στη ελληνική γλώσσα μεταφράστηκε από τον Χαραλ. Σιαδήμα και εκδόθηκε από το Υπουργείο Παιδείας το 1964 με χρήματα του Ο.Ο.Σ.Α. για τις ανάγκες των καθηγητών της Μέσης Εκπαίδευσης, αλλά αυτή η έκδοση δεν τράβηξε τότε την προσοχή των Ελλήνων εκπαιδευτικών. Τη δεκαετία του 2000 έχουμε την έκδοση του επίσης κλασικού έργου του G. Polya «Η μαθηματική ανακάλυψη. Κατανόηση, μάθηση και διδασκαλία του τρόπου επίλυσης προβλημάτων» σε επιμέλεια του Μ. Λάμπρου από τις εκδόσεις «Κάτοπτρο». Πρόκειται για ένα εξαιρετικό βιβλίο για το P.S., τόσο σε θεωρητικό, αλλά και σε πρακτικό επίπεδο, το οποίο όμως δεν έτυχε της ανάλογης προσοχής από το ευρύ κοινό, ίσως επειδή αφορά και θέματα που βρίσκονται λίγο μετά τα όρια της σχολικής διδακτέας ύλης για τα ελληνικά Γυμνάσια και Λύκεια. Επίσης, κατά τη χρονική περίοδο από το 1990 έως το 2010 εκδόθηκαν αρκετά βιβλία, εκτός των φροντιστηριακών, που αφορούσαν την επίλυση προβλημάτων διαγωνιστικού τύπου, τα οποία κάλυπταν τις ανάγκες για δύσκολα μαθηματικά προβλήματα. Αντίστοιχη σε παραγωγικότητα ήταν και η έρευνα από τη μεριά της Διδακτικής για το πώς οι μαθητές των σχολείων στην Ελλάδα είναι σε θέση να επιλύουν μαθηματικά προβλήματα.
Όμως, οι μελέτες και έρευνες για τη διαδικασία κατασκευής μαθηματικών προβλημάτων στη χώρα μας δεν έχουν να επιδείξουν κάτι το ουσιώδες. Το Ρ.Ρ. δεν έχει απασχολήσει σοβαρά τους ερευνητές της Διδακτικής. Για παράδειγμα, ούτε το κλασικό έργο του Ρ.Ρ. το «The Art of Problem Posing» των S. Brown και M. Walter, το οποίο εκδόθηκε το 1990, δεν έχει μεταφραστεί στην ελληνική γλώσσα, ούτε και άλλα βασικά κείμενα σχετικά με το Ρ.Ρ., έχουν μεταφραστεί ή έχουν σχολιαστεί ή έχουν αποτελέσει τη βάση για σχετικές μελέτες. Σε ερευνητικό επίπεδο ασχολείται η καθηγήτρια στο Μαθηματικό Τμήμα του Πανεπιστημίου Πατρών Γιάννα Μαμωνά, με δημοσιεύσεις όμως στην αγγλική κυρίως γλώσσα. Παρ’ ότι αρκετές εργασίες μεταπτυχιακού επιπέδου αφορούν τις διαδικασίες του P.S., μάλιστα και ορισμένες διδακτορικές διατριβές ασχολούνται με αυτές, για το Ρ.Ρ. δεν έχουμε κάτι ανάλογο. Μία αξιοπρόσεκτη δουλειά στο θέμα του Ρ.Ρ. διεξάγεται στην Κύπρο από τους Φιλίππου, Χρήστου και ομάδα συνεργατών τους, οι οποίοι και αυτοί δημοσιεύουν στην αγγλική κυρίως γλώσσα, αν και έχουν δώσει εργασίες και στην ελληνική γλώσσα.
Στη διεθνή έρευνα στη Διδακτική των Μαθηματικών και στην αντίστοιχη βιβλιογραφία τα δεδομένα είναι διαφορετικά. Για παράδειγμα στην μακρινή Αυστραλία ήδη από το 1991 το Αυστραλιανό Συμβούλιο Εκπαίδευσης στην διακήρυξή του με τίτλο «The National Statement on Mathematics for Australian Schools» (σελ. 39) προωθούσε τη χρήση ανοικτών προβλημάτων για τη  διδασκαλία των Μαθηματικών. Το ανοικτό πρόβλημα είναι μία μορφή Ρ.Ρ., η οποία σχετίζεται άμεσα με το P.S. (βλέπε Πούλος Α., 2007). Στις Η.Π.Α. το  Εθνικό Συμβούλιο Δασκάλων των Μαθηματικών (N.C.T.M.), η μεγαλύτερη Ένωση μαθηματικών διδασκόντων στην Πρωτοβάθμια και Δευτεροβάθμια εκπαίδευση, από τη δεκαετία του 1990, τονίζει ότι οι μαθητές των τάξεων από 9 έως 12 πρέπει να έχουν αποκομίσει κάποια εμπειρία διαμόρφωσης δικών τους προβλημάτων, μιας διαδικασίας η οποία είναι ο πυρήνας της έννοιας «ασχολούμαι με τα Μαθηματικά». (N.C.T.M., 1989, σελ. 138).
Η έννοια της διαδικασίας του Ρ.Ρ. τα τελευταία 30 χρόνια έχει αποσαφηνισθεί και έχει εμπλουτιστεί αρκετά. Για τους περισσότερους ερευνητές ο γενικός ορισμός του Silver (Silver, 1994) «το μαθηματικό Ρ.Ρ. αναφέρεται στην κατασκευή νέων προβλημάτων και στην ανασύνθεση ήδη κατασκευασμένων προβλημάτων» είναι επαρκής. Μία ανασκόπηση των πιθανών ορισμών που περικλείει το Ρ.Ρ. υπάρχει στο άρθρο (Stoyanova & Ellerton, 1996, σελ. 518). Σε αυτό δίνεται και ένας ορισμός, ο οποίος δίνει και μία άλλη διάσταση σε ήδη διατυπωμένους ορισμούς, «Το Ρ.Ρ. ορίζεται ως μία διαδικασία  κατά την οποία, με βάση τη μαθηματική εμπειρία, οι μαθητές κατασκευάζουν προσωπικές αναπαραστάσεις δεδομένων καταστάσεων και τις μετασχηματίζουν σε μαθηματικά προβλήματα που έχουν νόημα». Ακόμα και η αναμόρφωση και βελτίωση δεδομένων προβλημάτων ανήκει στη διαδικασία του Ρ.Ρ. αρκεί το παραγόμενο αποτέλεσμα να είναι ένα νέο πρόβλημα με νόημα και άξιο της προσοχής των λυτών. Ένα βασικό ερώτημα που απασχολεί τους ερευνητές της Διδακτικής των Μαθηματικών είναι αν υπάρχει ουσιώδης σχέση μεταξύ των ικανοτήτων για Ρ.S. και Ρ.Ρ. για το ίδιο άτομο και για το ίδιο είδος προβλημάτων. Ένα άλλο σημαντικό ερώτημα είναι πώς η εμπειρία στην επίλυση προβλημάτων επηρεάζει την ικανότητα για Ρ.Ρ.
Ακόμη και η μεταβολή των δεδομένων ενός προβλήματος αποτελεί μέρος της διαδικασίας του Ρ.Ρ. Η εμπειρία από τη διδασκαλία στην τάξη δείχνει ότι μπορούμε να μεταβάλουμε τα δεδομένα ενός μαθηματικού προβλήματος, χωρίς να μεταβληθεί  η δομή του, ούτε η διαδικασία της επίλυσής του. Επίσης, μπορεί σε κάποιες μεταβολές των δεδομένων, τα μαθηματικά εργαλεία της επίλυσης του νέου προβλήματος να παραμένουν τα ίδια. Όπως, υπάρχει το analogical problem-solving ( Voskoglou, 2003), έτσι υπάρχει και το Ρ.Ρ. κατ’  αναλογία, δηλαδή κατασκευή προβλήματος με ίδια δομή με κάποιο δεδομένο πρόβλημα. Σε άλλα όμως προβλήματα μία εσκεμμένη, ή μη, μεταβολή των δεδομένων μπορεί να «δυσκολέψει» ή να τροποποιήσει ριζικά το αρχικό πρόβλημα. Για παράδειγμα, το πρόβλημα «Για το τρίγωνο ΑΒΓ δίνονται α = 5, β = 6 και γωνία Α = π/3. Να βρεθούν οι άλλες γωνίες και πλευρές του τριγώνου» επιλύεται με τη βοήθεια του αποκαλούμενου νόμου των ημιτόνων. Όμως, η αλλαγή δεδομένου «δίνεται η γωνία Γ = π/3, αντί της γωνίας Α», απαιτεί τη χρήση άλλου τύπου, του νόμου των συνημιτόνων. Το ίδιο συμβαίνει και στη γενική μορφή των αντίστοιχων προβλημάτων. Αυτά απαιτούν ριζικά τροποποιημένη αντιμετώπιση, την ανακάλυψη του νόμου των ημιτόνων και των συνημιτόνων αντίστοιχα. Ένα χαρακτηριστικό λοιπόν, το οποίο παρουσιάζει παιδαγωγικό ενδιαφέρον κατά τη διαδικασία του Ρ.Ρ. είναι αν ο «κατασκευαστής» έχει επίγνωση πώς η αλλαγή έστω και ορισμένων δεδομένων ενός προβλήματος επηρεάζει τη μέθοδο επίλυσής του ή και ακόμα τη δομή του.
Ένα δεύτερο χαρακτηριστικό του Ρ.Ρ. είναι η αποκαλούμενη γενίκευση, την οποία πολλές φορές συναντάμε αυθόρμητα στην τάξη, κυρίως από μαθητές που έχουν αυξημένο ενδιαφέρον για τα Μαθηματικά. Για παράδειγμα, έστω ότι πραγματευόμαστε το πρόβλημα παραγωγής τύπων οι οποίοι να βρίσκουν Πυθαγόρειες τριάδες, δηλαδή σχέσεις μεταξύ των φυσικών αριθμών α, β και γ, έτσι ώστε να ισχύει η ισότητα α2 = β2 + γ2. Είναι αυθόρμητο το ερώτημα, αν είναι δυνατόν να υπάρχει τριάδα φυσικών αριθμών ώστε να ισχύει η ισότητα α3 = β3 + γ3 ή γενικότερα αν = βν + γν με ν φυσικό.
Ένα τρίτο χαρακτηριστικό του Ρ.Ρ. είναι η αρχή της δοκιμής, της μερικής ή πλήρους αμφισβήτησης, η οποία συνοψίζεται στη φράση “What-If-Not”. Για παράδειγμα, σε ένα πρόβλημα το οποίο καλείται να λύσει κάποιος, έχουμε το δεδομένο ότι «αν το τρίγωνο ΑΒΓ είναι ισοσκελές, τότε να αποδειχθεί ότι …». Σύμφωνα με την αρχή της δοκιμής, θέτουμε το ερώτημα, «αν το τρίγωνο ΑΒΓ δεν είναι ισοσκελές, τότε πώς επηρεάζονται τα ζητούμενα του προβλήματος;», ουσιαστικά θέτουμε για επίλυση ένα νέο πρόβλημα, βασισμένο στο αρχικό πρόβλημα.
Μία ενδιαφέρουσα περίπτωση Ρ.Ρ. είναι η κατασκευή αντιπαραδειγμάτων. Αν και αυτή φαίνεται εκ πρώτης όψεως ότι είναι διαδικασία του P.S., για παράδειγμα η εύρεση ενός αντιπαραδείγματος το οποίο να καταρρίπτει μία εικασία (π.χ. την περίφημη εικασία του Goldbach), η σύνθεση προβλημάτων τα οποία μπορεί να επιλυθούν μέσω αντιπαραδειγμάτων είναι μία πρακτική με  σαφή παιδαγωγικά αποτελέσματα. Θέτει ερωτήματα, τα οποία για να απαντηθούν απαιτούν ένα σύνολο τεχνικών, τεχνασμάτων και νοητικών κατασκευών. Ακόμα και σε σχολικό επίπεδο πολλά ερωτήματα που φαίνονται απλοϊκά δεν είναι καθόλου τέτοια, για παράδειγμα «το άθροισμα δύο περιοδικών συναρτήσεων με διαφορετική ή με την ίδια περίοδο, είναι περιοδική συνάρτηση;» είναι ένα ερώτημα (πρόβλημα) το οποίο αποδεικνύεται ότι έχει αρνητική απάντηση, με την έννοια ότι μπορούμε να κατασκευάσουμε περιοδικές συναρτήσεις των οποίων το άθροισμα δεν είναι περιοδική συνάρτηση.
Η αξίωση για συγκεκριμένους τρόπους επίλυσης ενός προβλήματος, με επιλεγμένες προϋποθέσεις και όρους ανήκει και αυτή στη διαδικασία του Ρ.Ρ. Για παράδειγμα, γνωρίζουμε ότι η εύρεση του μέσου ενός ευθύγραμμου τμήματος πραγματοποιείται με κανόνα και διαβήτη. Αν θέσουμε την αξίωση το μέσο να βρεθεί μόνο με διαβήτη (κατά-σκευή Mascheroni), τότε έχουμε θέσει ένα νέο πρόβλημα, πολύ δυσκολότερο από το αρχικό. Συνεπώς, ο περιορισμός του εύρους και του πλήθους των μαθηματικών εργαλείων (τύπων και θεωρημάτων) που απαιτούνται για την επίλυση ενός προβλήματος, ουσιαστικά θέτει ένα νέο πρόβλημα, το οποίο δεν είναι γνωστό αν είναι επιλύσιμο.
Στη συνέχεια παραθέτουμε μία ταξινόμηση των τρόπων με τους οποίους μπορούμε να εξετάσουμε το Ρ.Ρ. από την πλευρά της Διδακτικής των Μαθηματικών. Ο στόχος αυτής της ταξινόμησης είναι να δείξει πλευρές της διαδικασίας του Ρ.Ρ. και πώς αυτή σχετίζεται με ερωτήματα της Διδακτικής των Μαθηματικών.

Το Ρ.Ρ. και η ικανότητα τροποποίησης μαθηματικών προβλημάτων αποτελεί κριτήριο μαθηματικής δημιουργικότητας, (π.χ. το άρθρο των Singer, Pelczer, Voica, 2011).

Το Ρ.Ρ. αποτελεί μέτρο και κριτήριο κατανόησης και για τη διαδικασία της επίλυσης μαθηματικών προβλημάτων, (π.χ. το άρθρο των Koichu & Kontorovich, 2012).

Το Ρ.Ρ. αποτελεί κριτήριο κατανόησης μαθηματικών εννοιών, (π.χ. Stoyanova & Ellerton, 1996).
Το Ρ.Ρ. αποτελεί μέτρο αναγνώρισης μαθηματικών ταλέντων, (π.χ. ανακοίνωση των Younggi Choi και Jonghoon Dο στο 11ο Διεθνές Συνέδριο ICME του 2008, αν και αυτό επισημαίνεται και από τους Walter και Brown, 2005, σελίδα 170).

Το Ρ.Ρ. αποτελεί δραστηριότητα που σχετίζεται με την επίλυση προβλημάτων μαθηματικών διαγωνισμών, (π.χ. το άρθρο του Toh Tin Lam, 2012).
            Το Ρ.Ρ. αποτελεί ένα μέσον εμπλουτισμού της μαθηματικής εκπαίδευσης, (π.χ. εισήγηση της Μαμωνά στο 8ο Συνέδριο μαθηματικής Παιδείας, 1993).
Το Ρ.Ρ. είναι κριτήριο μελέτης των ατομικών διαφορών μέσω του χειρισμού αλγεβρικών και γεωμετρικών διαδικασιών για την κατασκευή ενός προβλήματος, (π.χ. το άρθρο του Oleksiy Yevdokimov,  2005).
Το Ρ.Ρ. χρησιμεύει στην εκπαίδευση μελλοντικών δασκάλων των Μαθηματικών και στην μετεκπαίδευση των εν ενεργεία δασκάλων των Μαθηματικών, (π.χ. Crespo, S. 2003).
Το Ρ.Ρ. και η αλληλεπίδραση του με τα τεχνολογικά εργαλεία, ή πώς η τεχνολογία εμπλουτίζει και διευκολύνει την κατασκευή νέων προβλημάτων, (π.χ. Christou, C. et al, 2005 και Abramovich & Cho 2012).

Εννοείται ότι όλες οι παραπάνω ταξινομήσεις αφορούν όχι μόνον τους μαθητές, αλλά τους φοιτητές και τους εκπαιδευόμενους δασκάλους των Μαθηματικών (θεσμός ο οποίος στη χώρα μας δεν είναι επίσημος), αλλά και τους εν ενεργεία δασκάλους των Μαθηματικών.
To Ρ.Ρ. ως αυτόνομη μαθησιακή διαδικασία, αλλά και ως διαδικασία, η οποία έχει τύχει της προσοχής και μελέτης των ερευνητών της Διδακτικής, διαμορφώνει βαθμιαία τη δική του ορολογία. Έτσι, ο όρος «κατάσταση - πλαίσιο» (απόδοση στην ελληνική του όρου task –environment), θεωρείται βασικός στο Ρ.Ρ. Με τον όρο αυτόν εννοούμε ένα αρχικό πεδίο αναφοράς με βάση το οποίο θα ενθαρρυνθούν οι ενδιαφερόμενοι για να θέσουν ερωτήματα και να κατασκευάσουν δικά τους προβλήματα. Ο όρος «σύνδρομο του σωστού» τον οποίο διατύπωσαν πρώτοι οι Brown και Walter (1990) και οι όροι «ανοικτό και κλειστό πρόβλημα» ανήκουν πλέον στην ορολογία που είναι σε μόνιμη χρήση στο Ρ.Ρ. Συνεπώς, ένα από τα καθήκοντα που έχουν οι ερευνητές της Διδακτικής των Μαθηματικών στη χώρα μας σε σχέση με τη διάδοση και προβολή του Ρ.Ρ. είναι και η σωστή και έγκυρη απόδοση στη γλώσσα μας όρων, όπως: Ημιδομημένο και πλήρως δομημένο πρόβλημα, problem finding, problem creating, problem envisaging, advanced mathematical thinking, creativity, prototypical problem, original problem, abstraction (όχι με την έννοια της απλής αφαιρετικής ικανότητας), Meaningful problem, problematic situation, cycling και άλλοι πολλοί.
Στο τελευταίο μέρος της εισήγησης για το Ρ.Ρ. δίνουμε ένα συγκεκριμένο παράδειγμα από έρευνα η οποία βρίσκεται σε φάση δημοσίευσης. Η έρευνα και το θεωρητικό μέρος της είναι προϊόν συνεργασίας μου με την Γιάννα Μαμωνά-Downs και τον Martin Downs. Πρόκειται για το γνωστό πρόβλημα της κινούμενης σκάλας, το οποίο συναντάμε σε πολλά βιβλία επίλυσης προβλημάτων. Για το πρόβλημα αυτό υπάρχει μία εκτενής βιβλιογραφία από την μεριά του P.S., αλλά από την μεριά του Ρ.Ρ. αυτή είναι σχετικά περιορισμένη. Η συγκεκριμένη έρευνα έχει διεξαχθεί σε ένα δείγμα 50 μαθητών της Β’ και Γ’ τάξης του Λυκείου, με εντελώς διαφορετικό πλαίσιο παρουσίασης και προϋποθέσεις. Εδώ την παρουσιάζουμε έτσι ώστε να έχει τη μορφή διαδικασίας κατασκευής μαθηματικών προβλημάτων, ως ένα παράδειγμα του Ρ.Ρ.

Δίνουμε σε φωτοτυπία στους μαθητές της τάξης την παρακάτω συνταγή για κρέπες.
Υλικά
·                     2 αβγά
·                     1 φλιτζάνι γάλα
·                     1/2 κουταλιά αλάτι
·                     1 φλιτζάνι αλεύρι
·                     2 κεσεδάκια λιωμένο βούτυρο ή μαργαρίνη ή λάδι.
Διαδικασία
1.    Χτυπάμε καλά τα αβγά, προσθέτουμε το γάλα, το αλάτι, το αλεύρι και το βούτυρο. Σκεπάζουμε και αφήνουμε για 30 λεπτά.
2.    Σε αντικολλητικό τηγάνι βάζουμε μια μικρή ποσότητα της ζύμης (ίσα να καλύπτει το τηγάνι), και μόλις ψηθεί από κάτω, τη γυρίζουμε.
3.    Αφήνουμε για λίγο και από την άλλη και τη βγάζουμε από το τηγάνι.
4.    Μπορούμε να τη γεμίσουμε με ότι θέλουμε και ή να τη φάμε έτσι ή να τη βάλουμε στο φούρνο για να λιώσει η γέμιση.
Από εδώ και πέρα αρχίζει η τέχνη της διδασκαλίας, να μπορούμε να μετατρέπουμε και την πλέον άσχετη συζήτηση σε διάλογο για τα Μαθηματικά, σε μαθηματική γνώση. Ξεκινάμε με τους μαθητές μία συζήτηση για το είδος των παρεμβάσεων που μπορούμε να κάνουμε, τόσο στην αντικατάσταση των υλικών της συνταγής, στις μεταβολές των ποσοτήτων των υλικών, στην αλλαγή της σειράς της διαδικασίας παρασκευής. Αφού ολοκληρωθεί αυτή η συζήτηση, οι μαθητές διαπιστώνουν ότι ο αριθμός των κάθε είδους αλλαγών και παρεμβάσεων που μπορούμε να κάνουμε ακόμα και σε μία απλή συνταγή μαγειρικής είναι αρκετές. Προσπαθούμε να δημιουργήσουμε ένα κλίμα συζήτησης και διαλόγου, από το οποίο να προκύπτουν δύο ουσιώδη συμπεράσματα για τη διδακτική μας παρέμβαση.
α) Αυτός που διαβάζει τη συνταγή και θέλει να κάνει παρεμβάσεις σε αυτήν, μετατρέπεται από απλός χρήστης, σε άτομο που παίρνει πρωτοβουλίες, σκέπτεται με ποιους τρόπους και με τι μέσα μπορεί να παρέμβει στα υλικά, στις ποσότητες των υλικών, στη διαδικασία παρασκευής τους, κ.α.
β) Οποιαδήποτε προσπάθεια αλλαγής και παρεμβάσεων δεν σημαίνει κατ’  ανάγκην ότι θα είναι από γευστική άποψη ευχάριστη. Όμως, κάποιες προσωπικές μας παρεμβάσεις μπορεί να έχουν ισοδύναμα ή και μερικές φορές καλύτερα γευστικά αποτελέσματα και αυτό είναι το κέρδος της όλης προσπάθειας.
Θεωρούμε ότι η παιδαγωγική διαδικασία που προαναφέραμε, η οποία μπορεί να διαρκέσει μία ολόκληρη διδακτική ώρα, δίνει έμπρακτα σε γενικές γραμμές το πλαίσιο στο οποίο μπορεί να κινηθεί μία δραστηριότητα του Ρ.Ρ.. Την επόμενη σχολική ώρα, δίνουμε το ακόλουθο μαθηματικό πρόβλημα, στο οποίο υπάρχει μόνο ένα βασικό ζητούμενο.
Το παρακάτω σχήμα περιγράφει μία σκάλα μήκους L, της οποίας το ένα άκρο στηρίζεται σε ένα κατακόρυφο τοίχο και το άλλο στο πάτωμα. Αν η σκάλα γλιστρά τίθεται το πρόβλημα της εύρεσης του γεωμετρικού τόπου του μέσου της σκάλας.


Επιλύουμε το πρόβλημα με τον τρόπο με τον οποίο έχουμε συνηθίσει τους μαθητές να αντιμετωπίζουν ένα πρόβλημα στη σχολική τάξη. Προς το τέλος της σχολικής ώρας υπενθυμίζουμε στους μαθητές όλη τη διαδικασία που είχε πραγματοποιηθεί στην τάξη σχετικά με τις παρεμβάσεις στη συνταγή για τις κρέπες και ζητάμε από αυτούς να σκεφτούν για το επόμενο μάθημα, τι είδους μεταβολές στα δεδομένα είναι δυνατόν να γίνουν στο πρόβλημα με την σκάλα, τι είδους νέα ερωτήματα και ζητούμενα μπορούν να διατυπωθούν και γενικά τους ζητάμε να λάβουν πρωτοβουλίες και να αναπτύξουν τη φαντασία τους. Με αφορμή το αρχικό πρόβλημα θέτουμε τον στόχο της παραγωγής νέων τροποποιημένων προβλημάτων, χωρίς να τους ζητάμε να τα επιλύσουν. Επειδή οι μαθητές δεν είναι εξοικειωμένοι με το πνεύμα του Ρ.Ρ., δίνουμε μία ή δύο νύξεις σχετικές με το τι είδους παρεμβάσεις θα μπορούσαν να γίνουν και τι επιπλέον ερωτήματα είναι δυνατό να διατυπωθούν. Ένα παράδειγμα είναι η ερώτηση «ποιό είναι το μέγιστο εμβαδόν του τριγώνου που ορίζεται από την κινούμενη σκάλα, τον τοίχο και το πάτωμα»;
Η εμπειρία, μικρή ή μεγάλη, που έχουμε ως διδάσκοντες με το Ρ.Ρ. μας κάνει προσεκτικούς και μας προξενεί άγχος σε δύο κυρίως σημεία κατά τη συζήτηση στην τάξη. Πρώτον, οι αναμενόμενες ερωτήσεις των μαθητών μας επιθυμούμε να είναι  κατά το δυνατόν συμβατές τόσο με τις γνώσεις τους, αλλά και με τις γνώσεις των υπόλοιπων συμμαθητών τους. Δεύτερον, η συζήτηση σχετικά με ποιες ερωτήσεις-προβλήματα πρέπει να επικεντρωθούν οι μαθητές να έχει κάποιες προϋποθέσεις, έτσι ώστε να μη διατυπώσουν ούτε απλοϊκά ερωτήματα, ούτε όμως και να ασχολούνται με προβλήματα που απαιτούν μία μεγάλη ποσότητα γνώσεων για να τα διαπραγματευτούν. Πρέπει να είμαστε ενήμεροι ότι η διαδικασία του Ρ.Ρ. απαιτεί μεγάλη προσοχή και κατάλληλη προετοιμασία, ώστε και να ενθαρρύνουμε τους μαθητές να θέτουν ερωτήματα και προβλήματα, αλλά να μην θεωρηθεί ως μία συζήτηση κατά την οποία ο καθένας μπορεί να λέει ότι θέλει χωρίς στόχο και σκέψη. Είναι συνηθισμένη πρακτική να έχουμε από πριν καταγράψει έναν κατάλογο με πιθανές ερωτήσεις και νέα προβλήματα που μπορεί να δώσει η διαδικασία του Ρ.Ρ. Οι μαθητές πρέπει βαθμιαία να αντιλαμβάνονται το Ρ.Ρ. ως μία διαδικασία κατά την οποία ο καθένας έχει την υποχρέωση (όχι το δικαίωμα) να θέσει μία τουλάχιστον ερευνητική υπόθεση εργασίας, ανεξάρτητα αν είναι σε θέση ο ίδιος να την απαντήσει ή να γνωρίζει με ποιόν τρόπο θα καταλήξει σε ένα βέβαιο συμπέρασμα.
Αυτό που κάνουμε στη συνέχεια είναι η καταγραφή ερωτημάτων από παλαιότερη εμπειρία στην τάξη, η οποία από μόνη της παράγει νέα ερωτήματα και ιδέες.
1.   Ποιος είναι ο γεωμετρικός τόπος ενός τυχαίου σημείου της σκάλας (όχι του μέσου της);
2.   Αν το άκρο Β κινείται με ταχύτητα v, το άκρο Α με τι ταχύτητα κινείται;
3.  Το εμβαδόν του τριγώνου ΟΑΒ είναι σταθερό ή μεταβλητό; Αν είναι μεταβλητό, σε ποια θέση αυτό θα γίνει μέγιστο;
4.    Αν το άκρο Α της σκάλας παραμένει σταθερό και το μήκος της σκάλας μεταβάλλεται με ταχύτητα v (επιμήκυνση) , τότε ποια είναι η απάντηση στα ερωτήματα 1, 2, και 3;
5.    Το ίδιο ερώτημα με το 4, αν το άκρο Β της σκάλας παραμένει σταθερό και η σκάλα επιμηκύνεται.
6.    Το ίδιο ερώτημα με το 4, αν το άκρο της σκάλας Α κινείται με σταθερή ταχύτητα v1 και το άκρο Β με σταθερή ταχύτητα v2.
7.    Αν υπάρχει ένα ημικύκλιο ακτίνας R με κέντρο (0, R), το οποίο ανήκει στο πρώτο τεταρτημόριο και η σκάλα κινουμένη εφάπτεται συνεχώς στο ημικύκλιο, όπως στο σχήμα που ακολουθεί, τότε ποιες είναι η απαντήσεις στα ερωτήματα 1, 2, 3, 4, 5 και 6; Ποιες είναι οι συνθήκες μεταξύ των αριθμών R και L μήκους σκάλας για να υπάρχει θετική απάντηση στα ερωτήματα;                           
8.   Αν υπάρχει ένα ημικύκλιο ακτίνας R με κέντρο (R, 0), το οποίο ανήκει στο πρώτο τεταρτημόριο και η σκάλα κινουμένη εφάπτεται συνεχώς στο ημικύκλιο, όπως στο σχήμα που ακολουθεί, τότε ποιες είναι η απαντήσεις στα ερωτήματα 1, 2, 3, 4, 5 και 6; Ποιες είναι οι συνθήκες μεταξύ των αριθμών R και L για να υπάρχει θετική απάντηση στα ερωτήματα;                           
9.   Αν το ημικύκλιο αντικατασταθεί με τετράγωνο πλευράς α, με κορυφές (0, 0), (0, α), (α, α) και (α, 0) και η σκάλα κινουμένη διέρχεται συνεχώς από το σημείο (α, α), όπως στο σχήμα που ακολουθεί, τότε ποιες είναι η απαντήσεις στα ερωτήματα 1, 2, 3, 4, 5 και 6; Ποιες είναι οι συνθήκες μεταξύ των αριθμών α και L για να υπάρχει θετική απάντηση στα ερωτήματα;                        
10.  Αν το τετράγωνο εμπόδιο αντικατασταθεί με ορθογώνιο κορυφών (0, 0), (α, 0), (α, β), (0, β) και η σκάλα κινουμένη διέρχεται συνεχώς από την κορυφή (α, β), όπως στο σχήμα που ακολουθεί, τότε ποιες είναι η απαντήσεις στα ερωτήματα 1, 2, 3, 4, 5 και 6; Ποιες είναι οι συνθήκες μεταξύ των αριθμών α, β και L για να υπάρχει θετική απάντηση στα ερωτήματα;                         
11. Αν το ορθογώνιο εμπόδιο αντικατασταθεί με ισοσκελές τρίγωνο κορυφών (0, 0), (α, β), (2α, 0) και η σκάλα κινουμένη διέρχεται συνεχώς από την κορυφή (α, β) του τριγώνου, όπως στο σχήμα που ακολουθεί, τότε ποιες είναι η απαντήσεις στα ερωτήματα 1, 2, 3, 4, 5 και 6; Ποιες είναι οι συνθήκες μεταξύ των αριθμών α, β και L για να υπάρχει θετική απάντηση στα ερωτήματα;
12.  Αν ο τοίχος στον οποίο στηρίζεται η σκάλα, δεν ήταν κάθετος στο πάτωμα, αλλά σχημάτιζε μία γωνία α < 90ο, τότε πώς επηρεάζονται οι απαντήσεις στα ερωτήματα από το 1 έως και το 11;
Προφανώς, ο αριθμός των ερωτημάτων που σχετίζονται με το αρχικό πρόβλημα της κινούμενης σκάλας, δεν είναι τόσο μικρός όσο φαίνεται αρχικά. Έτσι, μπορούν να διατυπωθούν και νέα ερωτήματα – προβλήματα όπως τα ακόλουθα.
·   Το εμβαδόν του τριγώνου ΟΑΒ είναι σταθερό ή μεταβλητό. Αν είναι μεταβλητό, να βρεθεί από τι εξαρτάται αυτό και πότε θα γίνει μέγιστο;
·  Το εμβαδόν του εγγεγραμμένου κύκλου του τριγώνου ΟΑΒ είναι σταθερό ή μεταβλητό και από ποιους παράγοντες εξαρτάται;
·  Το μήκος του τόξου ΑΒ που ορίζεται από τον περιγεγραμμένο κύκλο του τριγώνου ΟΑΒ είναι σταθερό ή μεταβλητό; Το ίδιο ερώτημα διατυπώνεται και για τα τόξα ΟΑ και ΟΒ.
·     Το άθροισμα ΟΑ + ΟΒ των πλευρών του ορθογωνίου τριγώνου ΟΑΒ είναι σταθερό ή μεταβλητό και από τι εξαρτάται;
Πρέπει να σημειώσουμε όλα αυτά τα ερωτήματα είναι ιδιαίτερα ελκυστικά για τους επιδέξιους χειριστές προγραμμάτων όπως το Geometer’s Sketchpad, ή το Geogebra. Πολλοί μαθητές είναι οι ίδιοι πολύ καλοί χειριστές τέτοιων προγραμμάτων. Η προσπάθεια απάντησης στα ερωτήματα μέσω τέτοιων προγραμμάτων, παράγει από μόνη της νέα ερωτήματα, τα οποία εμπλουτίζουν τον κατάλογο των ερωτημάτων μας και συνθέτουν νέα προβλήματα.
Σημειώνουμε ότι το πρόβλημα της κινούμενης σκάλας έχει τα ακόλουθα παιδαγωγικά πλεονεκτήματα, που το καθιστούν κατάλληλο για το Ρ.Ρ.
1. Η σκάλα είναι ένα οικείο αντικείμενο για τους μαθητές συμβατό με την εμπειρία τους.
2. Ο κάθε μαθητής ανάλογα με το βαθμό μαθηματικής ωριμότητας, ακόμα και μικρός να είναι αυτός, μπορεί να διατυπώσει κάποια ερωτήματα και να συνθέσει ένα τουλάχιστο σχετικό απλό πρόβλημα.
3. Θεωρούμε ότι είναι μία κλασική περίπτωση προβλήματος στο οποίο εύκολα εφαρμόζεται η στρατηγική της μερικής ή πλήρους αμφισβήτησης What-If-Not”.
3. Έχει βαθμούς ελευθερίας ώστε ο κάθε μαθητής να διατυπώσει προβλήματα αλγεβρικά, γεωμετρικά ή προβλήματα φυσικής, τα οποία θα αντιμετωπιστούν με μαθηματικά εργαλεία.
4. Στο πρόβλημα υπάρχουν μεταβλητές, οι οποίες ανάλογα με την τιμή τους παίζουν ουσιώδη ρόλο στη διατύπωση ερωτημάτων και στην επίλυση νέων προβλημάτων.

ΒΙΒΛΙΟΓΡΑΦΙΚΕΣ ΑΝΑΦΟΡΕΣ

·  Abramovich Sergei & Cho Eun Kyeong, (2012), Technology-Enabled Mathematical Problem Posing as Modeling. Journal of Mathematical Modelling and Application, Vol. 1, No.6, pp.22-32.
·   Brown Steven & Walter Marion (2005), The Art of Problem Posing. Hillsdale, N.J.: L. Erlbaum Associates. 3η έκδοση. Η 1η έκδοση το 1990.
·  Crespo, S., (2003), Learning to pose mathematical problems: Exploring changes in preservice teachers’ practices. Educational Studies in Mathematics, 52(3), pp.243-270.
·  Christou, C., & Mousoulides, N., & Pittalis, M. & Pitta-Pantazi, D. (2005), Problem Solving and Problem Posing in a Dynamic Geometry Environment. The Montana Mathematics Enthusiast, Vol. 2, Νo.2, pp. 125-143.
·         Einstein A., & Infeld L., (1938), The evolution of Physics.  N.Y., Simon and Schuster.
·   Gardner Antony, (1992), Creating Elementary Problems to Stimulate Thinking. Mathematics Competitions, Vol. 5, No1. pp. 58-67.
·        Kilpatrick J. (1987), Problem formulating: Where do good problem come from? In A. Schoenfeld, (Ed.), Cognitive science and Mathematics education, (pp.123-147), Hillsdale N.J.: Lawrence Erlbaum Associates.
·    Koichu Boris & Berman A., (2005), When do the gifted high school students use geometry to solve geometry problems? Journal of Secondary Gifted Education, 16(4), pp. 168-179.
·         Koichu Boris & Kontorovich Igor, (2012), Dissecting success stories on mathematical problem posing: A case of the Billiard Task. Educational Studies in Mathematics. No3, pp. 1-16.
·         Leung S.S. (1997). On the role of creative thinking in problem posing. Zentralblatt fur Didaktik der Mathematik, (ZDM), pp. 48-52.
·         Mamona-Downs, J., (1993), On Analysing Problem PosingIn I. Hirabayashi, N. Nohda, K. Shigematsu & Fou-Lai Lin (Eds.), Proceedings of the 17th Annual Conference of the P.M.E. International (III pp. 41-48), Tsukuba, Japan.
·         Mamona-Downs Joanna & Downs Martin, (2005), The identity of problem solving. The Journal of Mathematical Behavior, 24, pp. 385-401.
·      National Council of Teachers of Mathematics, (1989), Principles and standards for school Mathematics. Reston, VA:
·  Τσούκας Λ., Κύπρου Χ., Φιλίππου, Γ., (2005), Η ικανότητα κατασκευής και η ικανότητα επίλυσης μαθηματικού προβλήματος. Στο Η Διδακτική των Μαθηματικών ως Πεδίο Έρευνας στην Κοινωνία της Γνώσης, Κυνηγός Χ. (Επ.), Πρακτικά 1ου Συνεδρίου της ΕΝΕΔΙΜ, (σελ. 559-569), Εκδόσεις Ελληνικά Γράμματα, Αθήνα.
·        Polya George, (2001), Η μαθηματική ανακάλυψη, Τόμος 1ος. Εκδόσεις Κάτοπτρο, Αθήνα.
· Πούλος Ανδρέας, (2007), Τα ανοικτά προβλήματα ως μέσο διδασκαλίας για την επίλυση μαθηματικών προβλημάτων, στον τόμο «Θέματα Εισαγωγικής Επιμόρφωσης για νεοδιόριστους εκπαιδευτικούς», Επιμ. Ε. Μπότσαρη, έκδοση Παιδαγωγικού Ινστιτούτου, Αθήνα, σελ. 399-404.
·     Silver, E. A. & Mamona, J., (1989), Problem posing by middle school mathematics teachers. In C. Maher, G. Goldin & R. Davis (Eds.), Proceedings  of the 11th annual meeting of the North American Chapter of the International Group for the Psychology of Mathematics Education, (pp. 263-269). New Brunswick, NJ.
·     Silver E.A. & Mamona-Downs J. & Leung S. & Kenney P.A., (1996), Posing mathematical problems: An exploratory study. Journal for Research in Mathematics Education, 27(3), pp. 293-309.
 Singer, F. M., Pelczer, I., Voica, C., (2011), Problem posing and modification as a criterion of mathematical creativity. In T. Rowland & E. Swoboda (Eds.) Proceedings of the 7th Conference of the European Society for Research in Math Education (CERME 7), pp. 1133-1142. Poland: University of Rzeszów.
·       Stoyanova Elena, & Ellerton, Nerida, (1996). A framework for research into students’ problem posing. In P. Clarkson (Ed.), Technology in mathematics education (pp. 518-525). Proceedings of the 19th Annual Conference of the Mathematics Education Research Group of Australasia. Melbourne: MERGA.
·    Toh Tin Lam, (2012), The roles of Mathematics Competitions in Singapore Mathematics Education. Proceedings of 12th International Congress on Mathematical Education, COEX, Seoul, Korea.
· Yevdokimov Oleksiy, (2005), On development of students’ abilities in problem posing: a case of plane Geometry. Proceedings 4th Mediterranean Conference on Mathematics Education, pp.255- 267.
·        Younggi Choi & Jonghoon Do, (2008), Research on the characteristics of mathematically gifted students in Korea. Proceedings of the Discussing Group 9: Promoting Creativity for All Students in Mathematics Education. 11th I.C.M.E., Mexico. pp. 33-47.
·   Van Harpen, Xianwei, Y., & Sriraman, Bharath, (2013), Creativity and mathematical problem posing: an analysis of high school students' mathematical problem posing in China and the USA. Educational Studies in Mathematics. Vol. 82, 2, pp.201-213.
· Voskoglou, M. G., (2003). Analogical problem-solving and transfer. Proceedings 3th Mediterranean Conference on Mathematics Education, pp.295-303, Athens.
______________________________________________________________

Ένας πολύ χρήσιμος ιστότοπος που περιέχει θέματα μαθηματικών διαγωνισμών διαφόρων τύπων και επιπέδων δυσκολίας είναι ο  http://evripidis.freebsdgr.org/links/diagonismoi.html



Φωτογραφία της Ελληνικής Ολυμπιακής Ομάδας Μαθηματικών στην Αργεντινή 2012

_______________________________________________________________